Download as pdf or txt
Download as pdf or txt
You are on page 1of 55

06/01/2017, 17)40

Haemorrhoids are enlarged congested mucosal subepithelial vascular cushions found in the 3, 7 & 11 o’clock
positions that contribute to anal continence.

They are the commonest cause of haematochezia.


Factors contributing to haemorrhoids include a
low fibre diet
increased anal sphincter tone
obstruction of venous flow during pregnancy
portal hypertension
Haemorrhoids may be either
internal (origin above the dentate line)
external (origin below the dentate line)
can be classified into four grades
I - do not prolapse out of the anal canal
II - prolapse on defecation but reduce spontaneously
III - require manual reduction
IV - cannot be reduced
Haemorrhoids often only produce intermittent symptoms
Symptomatic episodes are often precipitated by constipation.
These episodes usually last from a few days to a few weeks.
Often they are completely asymptomatic between episodes.
Clinical features of piles include
bleeding after defecation (which may just stain the toilet paper or streak the faeces or if copious it
may splash around the lavatory pan)
faecal soiling
mucous discharge
pruritus ani
occasionally pain (thrombosis may cause acute severe anal pain occasionally necessitating hospital
admission)
rectal mass (grades II-IV)
Note haemorrhoids are often not palpable on digital examination because they empty with pressure from the
examining finger.
Proctoscopy is necessary to diagnose first or second degree piles where the piles are seen bulging into the lumen
as the proctoscope is withdrawn
A sigmoidoscopy is indicated if there is a history of bleeding or there are symptoms that are suspicious of
malignancy
Treatment includes
Conservative measures such as
improving diet
ensuring the perineum is dried and washed after defecation
digital replacement of prolapsed haemorrhoids can lead to symptomatic relief
local anaesthetic creams and ointments
glyceryl trinitrate 0.2-0.3% (unlicensed)
Outpatient interventions appropriate for grade I or II haemorrhoids include
sclerotherapy
rubber band ligation
photocoagulation
cryotherapy
botulinum toxin injection may help reduce spasm of the internal anal sphincter
Surgical options include
Lord's stretch procedure & haemorrhoidectomy

https://www.mcqbank.co.uk/note_manage_print.php Page 1 of 55
06/01/2017, 17)40

Q1 - A

Acute cholecystitis follows impaction of a stone in the cystic duct, which may cause continuous epigastric or right
upper quadrant pain, vomiting, fever, local peritonism, or a gallbladder mass.

The main difference from biliary colic is the inflammatory component (local peritonism, fever, raised white cell
count).
If the stone moves to the common bile duct jaundice may occur.
Repeated attacks of acute cholecystitis lead to chronic cholecystitis, in which the walls of the gallbladder become
thickened and scarred, and the gallbladder becomes shrivelled.
Murphy's sign is performed by asking the patient to breathe out and then gently placing the hand below the
costal margin on the right side at the mid-clavicular line (the approximate location of the gallbladder).
The patient is then asked to breathe in.
During inspiration, the abdominal contents are pushed downward and the gallbladder will meet the
examiner’s hand.
If the gallbladder is tender this will cause the patient to stop inspiring due to pain.
This is considered positive but a positive test also requires no pain on performing the manoeuvre on
the patient's left hand side.
Typically, it is positive in cholecystitis, but negative in choledocholithiasis and ascending cholangitis.

Q2 - C

Femoral hernias occur below the inguinal ligament, where abdominal contents pass through the femoral canal.

In 2005/6 there were 4,583 femoral hernia repairs in the NHS (c.f. 79,760 for inguinal hernia) in England & of
these, 49% were emergency repairs and 48% waiting list repairs.
Femoral hernia is 4 times commoner in women but in women, inguinal hernia is still more common than
femoral hernia.
The incidence is highest in middle-aged and elderly women, especially if parous.
It presents as a lump in the groin, lateral and inferior to the pubic tubercle (inguinal hernias are superior and
medial) but a large hernia may bulge over the inguinal ligament and make differential diagnosis difficult.
The lump is worse on coughing or straining and reduces in size or disappears when relaxed or supine.
The risk of strangulation in a femoral hernia is much higher than for an inguinal hernia and only 50% of patients
are aware of the hernia before strangulation.
A strangulated hernia presents as a red tender, tense irreducible lump in conjunction with signs of bowel
obstruction.

Q3 - E

Crohn's disease is a disorder of unknown aetiology that is characterised pathologically by involvement of all bowel
wall layers in a chronic inflammatory process with non-caseating granulomas.

The granulomatous inflammation most frequently affects the terminal ileum but it may affect any part of the
gastrointestinal tract and frequently affected areas are in discontinuity.
Possible local complications of include:
intestinal obstruction
haemorrhage
perforation with abscess
stricture formation
perianal abscess
fistulae to the bowel, bladder, vagina
increased risk of malignancy but less than that of ulcerative colitis
Common general complications include:
weight loss
anaemia

https://www.mcqbank.co.uk/note_manage_print.php Page 2 of 55
06/01/2017, 17)40

arthritis of the large joints


erythema nodosum
ocular problems (conjunctivitis, episcleritis, iritis)
sacroiliitis (this is unrelated to HLA B27)
gall stones, especially of the cholesterol type

Hypothermia & not hyperthermia is a cause of pancreatitis.

Acute pancreatitis is acute inflammation of the pancreas, releasing exocrine enzymes that cause autodigestion of
the organ.

Causes

Gallbladder disease and excess alcohol consumption account for most cases
“I GET SMASHED”
I Idiopathic
G Gallstones
E Ethanol
T Trauma
S Steroids
M Mumps
A Autoimmune (PAN)
S Scorpion Venom
H Hyperlipidaemia, Hypothermia, Hypercalcaemia
E ERCP
D Drugs:
AIDS drugs DDI and pentamidine
diuretics such as furosemide and hydrochlorothiazide
chemotherapeutic agents L-asparaginase and azathioprine
oestrogen

Presentation

Most commonly, presents as severe upper abdominal pain of sudden onset with vomiting with mild pyrexia
& tachycardia.
In severe cases: gross hypotension, pyrexia, tachypnoea, acute ascites, pleural effusions, body wall staining
around the umbilicus (Cullen's sign) or flanks (Grey Turner's sign) & hypoxaemia.

Investigations

Bloods
Serum amylase >4 x normal, amylase rises within a few hours after onset, returning to normal 3-5 days
after attack.
Imaging
Plain film to exclude other causes of an acute abdomen
CT (helps if diagnosis equivocal & in assessment of complications)
U/S (useful to detect gallstones)

Assessment of severity

https://www.mcqbank.co.uk/note_manage_print.php Page 3 of 55
06/01/2017, 17)40

Glasgow, Ranson & APACHE scores


≥ 3 of Modified Glasgow Criteria within 48hrs suggest severe Pancreatitis.
Mnemonic = PANCREAS
PaO2 <8kPa
Age >55yrs
Neutrophils (WBC >15x109/l)
Calcium <2mmol/l
Renal Function (Urea >16mmol/l)
Enzymes (LDH >600ih/l; AST>200iu/l)
Albumin <32g/l
Sugar (Glucose >10mmol/l)

Management

Mild cases
Manage on a general ward with pain relief (morphine is relatively contra-indicated because of possible
spastic effect on the sphincter of Oddi), IV fluids with nil by mouth, nasogastric tube if severe vomiting.
Severe cases
Treat in ITU or a high dependency unit.
Where there is evidence of significant pancreatic necrosis, IV antibiotics should be given,
Feed with enteral nutrition via a nasogastric tube
Surgery is only required where there is infection and necrosis.

Complications

Pancreatic necrosis - if infected, this trebles mortality risk


Acute fluid collections -the majority will resolve spontaneously
Pancreatic abscess is a collection of pus adjacent to the pancreas, presenting several months after an attack
Acute pseudo-cyst contains pancreatic juice in a wall of fibrous or granulation tissue
Pancreatic ascites occurs when a pseudo-cyst collapses into the peritoneal cavity or major pancreatic duct breaks
down and releases pancreatic juices into the peritoneal cavity.

Prognosis

5% mortality in mild cases


up to 30% mortality in severe cases

Q1 - G

A diverticulum is a herniation of mucosa through thickened colonic muscle.

Occur most commonly in the sigmoid and descending colon.


The main risk factors are age over 50 years and low dietary fibre.
About 50% of all people have diverticula by the time they are 50 years of age, and nearly 70% of all people
have diverticula by the time they are 80 years of age.
Presentation
Uncomplicated disease
Nonspecific abdominal complaints, e.g. lower abdominal pain, usually left-sided, bloating,
constipation
Complications
Diverticulitis
Presents with left lower quadrant pain and features of inflammation, such as pyrexia or
neutrophilia.
One third of patients who develop diverticulitis will develop further complications

https://www.mcqbank.co.uk/note_manage_print.php Page 4 of 55
06/01/2017, 17)40

(perforation, abscess, fistula, stricture/obstruction)


Haemorrhage
Common cause of lower gastrointestinal haemorrhage
Severe haemorrhage can arise in 3-5% of patients with diverticulosis
Presentation is usually abrupt painless bleeding with mild lower abdominal cramps, or the
urge to defecate followed by passage of a large amount of red or maroon blood or clots.
Melaena may occur but is uncommon.
Haemorrhage ceases spontaneously in 70-80% of patients.
Re-bleeding rates range from 22% to 38%.
Investigations
Colonoscopy, Barium enema, CT scanning
Mx
Advise a high-fibre diet
Hospital admission is required for people with severe diverticulitis or significant blood loss

Q2 - C

Angiodysplasia is a degenerative lesion of previously healthy blood vessels.

After diverticulosis, it is the second leading cause of lower GI bleeding in patients >60 years.
Found most commonly in the caecum and proximal ascending colon.
Clinical presentation is usually characterized by red-coloured stool, melaena, or haematochezia.
Bleeding is usually low grade but can be massive in approximately 15% of patients.

Q3 - E

Haemorrhoids are enlarged congested mucosal subepithelial vascular cushions found in the 3, 7 & 11 o’clock
positions that contribute to anal continence.

They are the commonest cause of haematochezia.


Factors contributing to haemorrhoids include a
low fibre diet
increased anal sphincter tone
obstruction of venous flow during pregnancy
portal hypertension
Haemorrhoids may be either
internal (origin above the dentate line)
external (origin below the dentate line)
can be classified into four grades
I - do not prolapse out of the anal canal
II - prolapse on defecation but reduce spontaneously
III - require manual reduction
IV - cannot be reduced
Haemorrhoids often only produce intermittent symptoms
Symptomatic episodes are often precipitated by constipation.
These episodes usually last from a few days to a few weeks.
Often they are completely asymptomatic between episodes.
Clinical features of piles include
bleeding after defecation (which may just stain the toilet paper or streak the faeces or if copious it
may splash around the lavatory pan)
faecal soiling
mucous discharge
pruritus ani
occasionally pain (thrombosis may cause acute severe anal pain occasionally necessitating hospital
admission)
rectal mass (grades II-IV)

https://www.mcqbank.co.uk/note_manage_print.php Page 5 of 55
06/01/2017, 17)40

Note haemorrhoids are often not palpable on digital examination because they empty with pressure from the
examining finger.
Proctoscopy is necessary to diagnose first or second degree piles where the piles are seen bulging into the lumen
as the proctoscope is withdrawn
A sigmoidoscopy is indicated if there is a history of bleeding or there are symptoms that are suspicious of
malignancy
Treatment includes
Conservative measures such as
improving diet
ensuring the perineum is dried and washed after defecation
digital replacement of prolapsed haemorrhoids can lead to symptomatic relief
local anaesthetic creams and ointments
glyceryl trinitrate 0.2-0.3% (unlicensed)
Outpatient interventions appropriate for grade I or II haemorrhoids include
sclerotherapy
rubber band ligation
photocoagulation
cryotherapy
botulinum toxin injection may help reduce spasm of the internal anal sphincter
Surgical options include
Lord's stretch procedure & haemorrhoidectomy

Q1 - G

Gilbert's disease is a benign, mildly symptomatic, non-haemolytic, unconjugated hyperbilirubinaemia.

The main abnormality is failure of uptake of albumin-bound bilirubin into hepatocytes.


Commoner in men.
Patients usually present with mild jaundice and malaise, symptoms persisting throughout life.
It is often a chance finding during screening, and has no significant morbidity or mortality.
Diagnosis is based on hyperbilirubinaemia with normal plasma bile acids,
An increase in unconjugated plasma bilirubin following a low energy diet or intravenous injection of
nicotinic acid
Levels reach more than twice the basal level with a peak after 2 to 3 hours as the nicotinic acid
causes a red cell haemolysis
Liver function tests are essentially normal except for a raised serum bilirubin.
The total plasma bilirubin can be as high as 80μmol/l.
There is no bilirubinuria
Unlike haemolytic anaemia:
normal reticulocyte count
urinary urobilinogen is not increased
Liver biopsy is not indicated.
No treatment is required.

Q2 - B

Budd-Chiari syndrome is an uncommon condition caused by obstruction to hepatic venous outflow.

Occurs most often in patients with underlying thrombotic diathesis, including:


myeloproliferative disorders, such as polycythemia vera and paroxysmal nocturnal haemoglobinuria
pregnancy
tumours
chronic inflammatory diseases
clotting disorders

https://www.mcqbank.co.uk/note_manage_print.php Page 6 of 55
06/01/2017, 17)40

infections
May be slightly more common in women and the presents more often in the 3rd -4th decade of life.
It is characterised by hepatomegaly, ascites, and abdominal pain.
Liver function tests may be mildly elevated.
Ascitic fluid usually has high protein content.
CT scan or MRI may show a prominent caudate lobe
Doppler ultrasound may help to exclude hepatic venous or inferior vena cava thrombosis.
Liver biopsy often shows centrilobular congestion.
Management involves treating any underlying haematological condition
Ascites should be managed with diuretics plus fluid and salt restriction.
Surgical decompression of liver is performed in cases of persistent congestion e.g. via transjugular
intrahepatic portosystemic shunt (TIPS)
Balloon angioplasty/stent is indicated for inferior vena caval web and sometimes in hepatic vein
thrombosis if the affected length of vein is not extensive.
Liver transplantation may be appropriate if there is decompensated liver cirrhosis.

Q3 - D

Pancreatic carcinoma is the third most common cancer of the gastrointestinal tract and the sixth most common
cause of cancer deaths.

Risk factors include


smoking
chronic pancreatitis
possibly secondary to alcohol consumption
high fat diet
diabetes mellitus
occupational exposure to coal gas and aluminium
It is slightly more common in women than men and median age at diagnosis is 65-69 years.
70% of pancreatic tumours occur in the head of the pancreas.
Progress is insidious.
There is usually early regional lymph node involvement and spread to the liver and the tumour may
also metastasize to bone and lung.
Weight loss is the commonest symptom.
Pain is usually epigastric and radiates to the back (but tumours of head of pancreas are usually
painless to start with i.e. painless jaundice).
Jaundice with pruritus if the tumour affects head of the pancreas and obstructs the biliary system.
There may also be ascites with a knobbly liver in 30%.
On examination the gallbladder may be palpable
Courvoisier's law or sign states that in the presence of a palpable gall bladder, painless jaundice is
unlikely to be caused by gall stones.
This is because gall stones are formed over a longer period of time, and this results in a shrunken,
fibrotic gall bladder which does not distend easily.
Therefore the gall bladder is more often enlarged in pathologies that cause obstruction of the biliary
tree over a shorter period of time such as pancreatic malignancy.
Surgery (radical pancreaticoduodenectomy or Whipple’s operation) offers the only possibility of cure but is only
suitable in <10% who present.
Mean survival <6mths.

Q1 - G

Nutritional support is important for patients who have major burns in order to minimize protein catabolism, total
body weight loss and glucose intolerance.

https://www.mcqbank.co.uk/note_manage_print.php Page 7 of 55
06/01/2017, 17)40

The catabolic response after a burn injury may be severe, and the basal metabolic rate may increase to more than
twice the baseline.
Nasojejunal feeding is preferred to nasogastric feeding as gastric dilatation and poor gastric emptying can occur.
TPN is not used in order to prevent gut bacterial translocation.

Q2 - B

Most patients with an acute flare up of their Crohn’s disease can be managed initially with normal diet and supplements.
Some may require only-liquid diets, others may need NGT feeding. If this wasn’t successful, a discussion regarding
starting TPN would need to be had amongst the clinical team.

Indications for TPN in Crohn's disease:

Short-term use: Patients with active inflammation and severe malnutrition and those with fistulae (given
preoperatively).
Long-term use: Patients who have had extensive intestinal resection, resulting in short bowel syndrome.

Q3 - D

A feeding jejunostomy is usually formed at surgery to allow early postoperative feeding and nutritional support.

A tumour marker is a substance found in the blood, urine, or body tissues that can be elevated in cancer, among other
tissue types.

Tumour markers can be produced directly by the tumour or by non-tumour cells as a response to the presence of a
tumour.
It should be noted that tumour markers usually have a low specificity (i.e lots of false +ves)

Common tumour markers:

Tumour Marker Tumour


CA 125 Ovarian cancer
CA 19-9 Pancreatic cancer
CA 15-3 Breast cancer
Prostate specific antigen Prostatic carcinoma
Hepatocellular carcinoma
Alpha-feto protein
Teratoma
Carcinoembryonic antigen Colorectal cancer

This patient has an anal fissure. Sclerotherapy is used in the treatment of haemorrhoids. It involves the injection of a
sclerosing agent (such as phenol) into the haemorrhoid which causes the vein walls to collapse and the haemorrhoids to
shrivel up.

An anal fissure is a crack in the wall of the anal mucosa so that the circular muscle layer is exposed.

https://www.mcqbank.co.uk/note_manage_print.php Page 8 of 55
06/01/2017, 17)40

The tear usually occurs in the midline posteriorly.


Affects men and women equally with peak incidence is in the 20-30 year old age group.
Usually due to passage of a large stool after a period of constipation but may also be secondary to Crohn's
disease.
Patient complains of pain after defecation and the passing of stools may cause sphincter spasm and pain
which may persist for up to an hour afterwards.
There is often fresh rectal bleeding and the patient complains of 'bright red blood on the paper'.
The patient may develop a fear of defecation which in turn aggravates the constipation.
On examination, there is a longitudinal tear that nearly always occurs directly posteriorly and in the midline.
A sentinel pile (torn, bunched-up strip of mucosa at the base of the fissure) may be seen if the anus is
inspected with the buttocks gently parted.
The severity of the pain and muscle spasm may preclude digital examination and necessitate examination
under general anaesthesia.
Conservative management includes
improving diet
topical glyceryl trinitrate
botulinum toxin injections (relaxes the anal sphincter to allow the fissure)
Conservative management is inappropriate if a sentinel tag or fibroepithelial polyp is present in which case the
patient will require surgical management which includes internal sphincterotomy & manual dilatation.

Q1 - D

This patient has gallstone disease.

The clinical stages of cholelithiasis (gallstones in the bladder) are:

Asymptomatic
Symptomatic (biliary colic)
Complicated (e.g., cholecystitis, choledocholithiasis, cholangitis)

Most gallstones are asymptomatic.

Classically, biliary colic is described as episodic pain in the right upper quadrant that radiates to the right
shoulder or back.
It begins postprandially (usually within an hour) and may last from 1-5 hours.
It is caused by contraction of the gallbladder (in response to a fatty meal) against an obstructing gallstone
(or sludge) in the cystic duct.
This leads to increased pressure within the gallbladder and pain.
The pain is often described as intense and dull and typically subsides after several hours, when the
gallbladder stops contracting and the stone falls back into the gallbladder.
Associated symptoms may include diaphoresis, nausea, and vomiting.

Q2 - D

This patient has an ovarian cyst which is a fluid-filled sac in an ovary. The increase in urinary frequency can arise
from large cysts putting pressure on the bladder.

Ovarian cysts are usually asymptomatic and are often an incidental finding during ultrasonography performed
for other reasons.
Lower abdominal pain is the most common symptom reported, with pain being sharp, intermittent, sudden, and
severe.
A sudden onset of abdominal pain may suggest cyst rupture but more serious aetiologies, including adnexal

https://www.mcqbank.co.uk/note_manage_print.php Page 9 of 55
06/01/2017, 17)40

torsion, perforated viscus, ectopic pregnancy, or appendicitis, must be considered.


Strenuous activities, such as exercise or sexual intercourse, may precede torsion or rupture.
Nausea and/or vomiting are nonspecific symptoms often associated with any of the above presentations in the
differential diagnosis of lower abdominal pain.
Ultrasound is the primary imaging tool for a patient thought to have an ovarian cyst.
It is most helpful when transvaginal and transabdominal modalities are used together.
Transabdominal ultrasonography requires the patient to have a full bladder, as opposed to transvaginal
ultrasonography.

Q3 – B

This patient most likely has a perforated peptic ulcer & a pneumoperitoneum which refers to the presence of air within
the peritoneal cavity.

The most common cause is a perforation of the abdominal viscus - most commonly, a perforated ulcer (as in
this case), although a pneumoperitoneum may occur as a result of perforation of any part of the bowel.
The exception is a perforated appendix, which seldom causes a pneumoperitoneum.
The presence of a pneumoperitoneum does not, however, always imply a perforation, because a number of
other conditions are associated with pneumoperitoneum:
infection of the peritoneal cavity with gas-forming organisms
rupture of an adjacent abscess or extension from the chest — Pneumomediastinum and bronchopleural
fistula
common after abdominal surgery
usually resolves 3-6 days after surgery, although it may persist for as long as 24 days after surgery
introduction of air through the female genital tract = most common cause of a spontaneous
pneumoperitoneum
Likewise, not every bowel perforation results in a pneumoperitoneum - some perforations seal over, allowing
little gas to escape.
An erect chest x-ray will show gas under the diaphragm.

Q1 - C

Murphy's sign is performed by asking the patient to breathe out and then gently placing the hand below the costal
margin on the right side at the mid-clavicular line (the approximate location of the gallbladder).

The patient is then asked to breathe in.


During inspiration, the abdominal contents are pushed downward and the gallbladder will meet the
examiner’s hand.
If the gallbladder is tender this will cause the patient to stop inspiring due to pain.
This is considered positive but a positive test also requires no pain on performing the manoeuvre on the
patient's left hand side.
Typically, it is positive in cholecystitis, but negative in choledocholithiasis and ascending cholangitis.

https://www.mcqbank.co.uk/note_manage_print.php Page 10 of 55
06/01/2017, 17)40

Q2 - G

Courvoisier's law or sign states that in the presence of a palpable gall bladder, painless jaundice is unlikely to be
caused by gall stones.

This is because gall stones are formed over a longer period of time, and this results in a shrunken, fibrotic gall
bladder which does not distend easily.
Therefore the gall bladder is more often enlarged in pathologies that cause obstruction of the biliary tree over a
shorter period of time such as pancreatic malignancy.

Q3- A

Rovsing's sign is positive when palpation of the lower left quadrant of a person's abdomen results in more pain in
the right lower quadrant indicating the patient may have appendicitis.

Colorectal carcinoma

3rd most commonly diagnosed cancer in UK (after Breast & lung).


Colon cancer is equally common in men and women, but rectal cancer is more common in men.
Risks factors for colorectal carcinoma include:
Age > 40 yr
Lifestyle - Dietary factors
Genetic factors:
familial adenomatous polyposis
Gardner's and Turcot's syndromes
familial colorectal cancer syndrome (Lynch I)
hereditary adenocarcinomatosis syndrome (Lynch II)
family history of colorectal carcinoma
Colorectal disease:
inflammatory bowel disease
benign colorectal polyps
personal history of colorectal cancer
pelvic irradiation
Other:
Hormone replacement therapy - reduces the chance of colorectal cancer
The distribution of colorectal cancer is:
45% rectum
25% sigmoid colon
15% caecum and ascending colon
10% transverse colon
5% descending colon
The presentation depends on the site of the cancer:
Right colon cancers
weight loss, anaemia, occult bleeding, mass in right iliac fossa
disease more likely to be advanced at presentation.
Left colon cancers
colicky pain, rectal bleeding, bowel obstruction, tenesmus, mass in left iliac fossa, early change
in bowel habit
less advanced disease at presentation.
Investigations
Colonoscopy is the gold standard for diagnosis of colorectal cancer.
Computed tomographic (CT) colonography is an effective, safe method for examining the colon and

https://www.mcqbank.co.uk/note_manage_print.php Page 11 of 55
06/01/2017, 17)40

rectum to detect abnormalities such as polyps and cancer.


Liver ultrasound, (occasionally intrarectal ultrasound) and CT or magnetic resonance imaging (MRI)
are useful in staging. MRI is more specific than CT in showing liver metastases.
2015 Cancer referral guidelines (see https://www.nice.org.uk/guidance/ng12):
REFER 2ww for colorectal cancer if:
≥40 with unexplained WEIGHT LOSS and ABDOMINAL PAIN.
≥50 with unexplained RECTAL BLEEDING.
≥60 with:
IRON-DEFICIENCY ANAEMIA.
CHANGE IN BOWEL HABIT.
+ve FAECAL OCCULT BLOOD.
CONSIDER 2ww referral for colorectal cancer in:
People with a rectal or abdominal mass.
Adults <50 with rectal bleeding AND any of the following unexplained symptoms:
Abdominal pain.
Change in bowel habit.
Weight loss.
Iron deficiency anaemia.
Offer faecal occult blood testing in adults without rectal bleeding who are:
≥50 with unexplained:
Abdominal pain.
Weight loss.
<60 with:
Changes in bowel habit.
Iron deficiency anaemia.
≥60 with:
Anaemia even in the absence of iron deficiency.
Staging
The Dukes' staging classification is now gradually being replaced by the tumour/node/metastases (TNM)
classification.
Management
Surgery remains the definitive treatment for apparently localised colorectal cancer.
Both radiotherapy and chemotherapy can improve survival rates after potentially curative surgery, and
chemotherapy prolongs overall survival in patients with advanced disease.

This gentleman has a post-operative paralytic ileus is defined as ileus of the gut persisting for more than 3 days.

Ileus occurs from hypomotility of the gastrointestinal tract in the absence of a mechanical bowel
obstruction.
Causes include:
sepsis
drugs
opioids, antacids, amitriptyline, chlorpromazine
metabolic
low potassium, magnesium, or sodium levels; anaemia; hyposmolality
myocardial infarction
pneumonia
trauma
fractured ribs, fractured spine
biliary and renal colic
head injury and neurosurgical procedures

https://www.mcqbank.co.uk/note_manage_print.php Page 12 of 55
06/01/2017, 17)40

intra-abdominal inflammation and peritonitis


retroperitoneal haematomas
Patients typically present with
vague, mild abdominal pain and bloating
They may report nausea, vomiting, and poor appetite.
Abdominal cramping is usually not present.
Patients may or may not continue to pass flatus and stool.
On examination, patients may have distended and tympanic abdomens, depending on the degree of abdominal
and bowel distension.
The abdomen may be tender.
A distinguishing feature is absent or hypoactive bowel sounds unlike the high-pitched sound of
obstruction.
The silent abdomen of ileus reveals no discernible peristalsis or succussion splash.
On plain abdominal radiographs, ileus appears as copious gas dilatation of the small intestine and colon.
Most cases resolve with watchful waiting and supportive treatment.
Keep patients nil by mouth and well hydrated.
For patients with vomiting and distension, use of a nasogastric tube provides symptomatic relief.

Gilbert's disease is a benign, mildly symptomatic, non-haemolytic, unconjugated hyperbilirubinaemia.

The main abnormality is failure of uptake of albumin-bound bilirubin into hepatocytes.


Commoner in men.
Patients usually present with mild jaundice and malaise, symptoms persisting throughout life.
It is often a chance finding during screening, and has no significant morbidity or mortality.
Diagnosis is based on hyperbilirubinaemia with normal plasma bile acids,
An increase in unconjugated plasma bilirubin following a low energy diet or intravenous injection of
nicotinic acid
Levels reach more than twice the basal level with a peak after 2 to 3 hours as the nicotinic acid
causes a red cell haemolysis
Liver function tests are essentially normal except for a raised serum bilirubin.
The total plasma bilirubin can be as high as 80μmol/l.
There is no bilirubinuria
Unlike haemolytic anaemia:
normal reticulocyte count
urinary urobilinogen is not increased
Liver biopsy is not indicated.
No treatment is required.

Enterobacter are common Gram-negative, facultatively anaerobic, rod-shaped bacteria that are found in the
environment and also in the gastrointestinal tract.

Several strains are pathogenic, the most common being E. cloacae and E. aerogenes, which can cause opportunistic
infections in immunocompromised patients. The urinary and respiratory tracts are the most common sites of infection..

Infective causes of bloody diarrhoea include:

https://www.mcqbank.co.uk/note_manage_print.php Page 13 of 55
06/01/2017, 17)40

Salmonella
Shigella
Campylobacter jejuni
Yersinia enterocolitica
E. coli
Entamoeba histolytica

Q1 - E

A Perianal haematoma is an acutely painful condition that often has an onset after straining at stool.

The haematoma is caused by rupture of a blood vessel beneath the anal skin.
On examination, there is a blue-black bulge in the skin near the margin of the anus.
Sometimes this condition is described as a 'thrombosed external pile' but it is not related at all to
haemorrhoids.
Generally the perianal haematoma resolves over a few days and the patient requires only oral analgesia.
If the pain becomes intolerable then it is possible to incise and drain the haematoma under anaesthesia.

Q2 - A

An anal fissure is a crack in the wall of the anal mucosa so that the circular muscle layer is exposed.

The tear usually occurs in the midline posteriorly.


Affects men and women equally with peak incidence is in the 20-30 year old age group.
Usually due to passage of a large stool after a period of constipation but may also be secondary to Crohn's
disease.
Patient complains of pain after defecation and the passing of stools may cause sphincter spasm and pain
which may persist for up to an hour afterwards.
There is often fresh rectal bleeding and the patient complains of 'bright red blood on the paper'.
The patient may develop a fear of defecation which in turn aggravates the constipation.
On examination, there is a longitudinal tear that nearly always occurs directly posteriorly and in the midline.
A sentinel pile (torn, bunched-up strip of mucosa at the base of the fissure) may be seen if the anus is
inspected with the buttocks gently parted.
The severity of the pain and muscle spasm may preclude digital examination and necessitate examination
under general anaesthesia.
Conservative management includes
improving diet
topical glyceryl trinitrate
botulinum toxin injections (relaxes the anal sphincter to allow the fissure)
Conservative management is inappropriate if a sentinel tag or fibroepithelial polyp is present in which case the
patient will require surgical management which includes internal sphincterotomy & manual dilatation.

Q3 - H

Haemorrhoids are enlarged congested mucosal subepithelial vascular cushions found in the 3, 7 & 11 o’clock
positions that contribute to anal continence.

They are the commonest cause of haematochezia.


Factors contributing to haemorrhoids include a
low fibre diet
increased anal sphincter tone
obstruction of venous flow during pregnancy

https://www.mcqbank.co.uk/note_manage_print.php Page 14 of 55
06/01/2017, 17)40

portal hypertension
Haemorrhoids may be either
internal (origin above the dentate line)
external (origin below the dentate line)
can be classified into four grades
I - do not prolapse out of the anal canal
II - prolapse on defecation but reduce spontaneously
III - require manual reduction
IV - cannot be reduced
Haemorrhoids often only produce intermittent symptoms
Symptomatic episodes are often precipitated by constipation.
These episodes usually last from a few days to a few weeks.
Often they are completely asymptomatic between episodes.
Clinical features of piles include
bleeding after defecation (which may just stain the toilet paper or streak the faeces or if copious it
may splash around the lavatory pan)
faecal soiling
mucous discharge
pruritus ani
occasionally pain (thrombosis may cause acute severe anal pain occasionally necessitating hospital
admission)
rectal mass (grades II-IV)
Note haemorrhoids are often not palpable on digital examination because they empty with pressure from the
examining finger.
Proctoscopy is necessary to diagnose first or second degree piles where the piles are seen bulging into the lumen
as the proctoscope is withdrawn
A sigmoidoscopy is indicated if there is a history of bleeding or there are symptoms that are suspicious of
malignancy
Treatment includes
Conservative measures such as
improving diet
ensuring the perineum is dried and washed after defecation
digital replacement of prolapsed haemorrhoids can lead to symptomatic relief
local anaesthetic creams and ointments
glyceryl trinitrate 0.2-0.3% (unlicensed)
Outpatient interventions appropriate for grade I or II haemorrhoids include
sclerotherapy
rubber band ligation
photocoagulation
cryotherapy
botulinum toxin injection may help reduce spasm of the internal anal sphincter
Surgical options include
Lord's stretch procedure & haemorrhoidectomy

Irritable bowel syndrome (IBS) is a chronic, relapsing disorder characterised by abdominal discomfort, which may be
associated with defecation and/or accompanied by a change in bowel habit.

IBS is the most common functional GI disorder - there is no structural lesion, but it may be explained by
abnormal smooth muscle activity ± visceral hypersensitivity.
IBS may present at any age (peak prevalence in 30s and 40s - female predominance is most obvious in the 3rd
decade and declines afterwards).
NICE (http://www.nice.org.uk/guidance/CG61) states that a diagnosis of IBS should be considered only if the

https://www.mcqbank.co.uk/note_manage_print.php Page 15 of 55
06/01/2017, 17)40

person has abdominal pain or discomfort that is either relieved by defecation or associated with altered
bowel frequency or stool form.
This should be accompanied by at least two of the following four symptoms:
altered stool passage (straining, urgency, incomplete evacuation)
abdominal bloating (more common in women than men), distension, tension or hardness
symptoms made worse by eating
passage of mucus
Other features such as lethargy, nausea, backache and bladder symptoms are common in people with IBS, and
may be used to support the diagnosis.
Investigations should be limited to those that are likely to exclude serious alternative diagnoses and when
negative support a positive diagnosis of IBS.
The most important differential diagnoses are coeliac disease, colorectal carcinoma and colitis.
Management:
General dietary advice
Have regular meals - i.e. avoid long gaps between meals and don't rush them.
Drink plenty of fluids (at least 8 cups per day) but restrict tea/coffee to 3 cups or so per day.
Reduce intake of alcohol and fizzy drinks.
Consider limiting high-fibre foods (e.g. wholemeal flour or bran), and resistant starches (often in
processed or recooked foods, and fresh fruits - limit to 3 portions per day).
For diarrhoea - avoid sorbitol.
For wind - consider increasing oats and linseeds (one tablespoon/day).
Food intolerance is common with IBS although true allergies are rare - suggest omitting any known food triggers,
but a formal exclusion diet needs the support of a dietician.
Drug therapy is directed towards symptomatic relief:
Antispasmodics especially mebeverine through their anti-muscarinic actions may relieve pain by
moderating smooth-muscle contractions.
Peppermint oil before meals may be of benefit with colonic spasm and symptoms of bloating
NICE suggest:
Laxatives should be considered for the treatment of constipation in people with IBS, but people should be
discouraged from taking lactulose.
Loperamide should be the first choice of antimotility agent for diarrhoea in people with IBS.
Tricyclic antidepressants (TCAs) as second-line treatment for people with IBS if laxatives, loperamide or
antispasmodics have not helped.
Selective serotonin reuptake inhibitors (SSRIs) should be considered for people with IBS only if TCAs
have been shown to be ineffective.
More than 50% will continue to have symptoms after 5 years.

Digoxin & organophosphate poisoning cause diarrhoea.

Calcium has some depressive effect on the CNS that can lead to coma, confusion, somnolence, weakness.
Gastrointestinal symptoms are related to the depressive action of hypercalcemia on the autonomic nervous system and
resulting smooth-muscle hypotonicity. Constipation is aggravated by dehydration that accompanies hypercalcemia
(high concentration of calcium in the blood promotes renal clearance through osmotic diuresis leading to polyuria).

The following are metabolic causes of constipation:

Hypothyroidism
Hypercalcaemia

https://www.mcqbank.co.uk/note_manage_print.php Page 16 of 55
06/01/2017, 17)40

Hypokalaemia
Porphyria
Lead poisoning

Given his ethnicity, hepatocellular carcinoma is the most likely diagnosis. The mass is related to the liver as can be
ascertained from the inability to get between the mass and the right costal margin. The signs in the chest are due to the
upward displacement of the diaphragm by tumour or to a pleural effusion following malignant infiltration of the pleura.

Hepatocellular carcinoma (HCC) is a primary malignancy of the liver.

The incidence of HCC is highest in Asia and Africa, where the endemic high prevalence of hepatitis B and
hepatitis C strongly predisposes to the development of chronic liver disease and subsequent development of
hepatocellular carcinoma.
Other causes include:
cirrhosis of any cause but especially alpha-1 antitrypsin deficiency, haemochromatosis, & primary
biliary cirrhosis
drugs such as anabolic steroids & the contraceptive pill
toxins such as aflatoxin (a toxin produced by species of Aspergillus which grows on stored grains and
peanuts)
parasitic infestations (schistosomiasis, echinococcus, clonorchis sinensis)
The age of onset is usually in the 7th decade in Western societies but in endemic areas it is most often in the 4th
or 5th decade.
In those societies the male preponderance is about 8:1 but in Western societies it is around 3:1.
The clinical manifestations are seldom characteristic and are often masked by those related to the background of
cirrhosis or chronic hepatitis.
HCC should be suspected if a patient with cirrhosis deteriorates without obvious reason or if ascites
fails to resolve despite adequate treatment.
Only 25% of tumours or less are amenable to surgery.
The rest are too advanced for resection at the time of diagnosis or there are other factors that mitigate
against surgery such as liver failure.
Overall the 5 years survival after resection is only 40%.
Other treatments are essentially ineffective.

Haemochromatosis is the abnormal accumulation of iron in the liver, heart, pancreas, pituitary, joints, and skin.

It is the most common autosomal recessive genetic disorder affecting 1:300 persons, mostly of northern
European origin.
The gene responsible is called HFE found on chromosome 6.
Early symptoms include severe fatigue, impotence & arthralgia.
Later, patients may experience skin bronzing or hyperpigmentation reflecting a combination of iron
deposition and melanin.
The classic triad of cirrhosis, diabetes mellitus, and skin pigmentation occurs late in the disease, when
total iron body content is 20 grams (ie, >5x normal).
Hepatocellular carcinoma is one of the most serious complications of haemochromatosis.
Other presentations include:
arthralgia

https://www.mcqbank.co.uk/note_manage_print.php Page 17 of 55
06/01/2017, 17)40

cardiomyopathy (congestive heart failure or arrhythmias)


hypogonadism due to pituitary involvement by iron deposition (decreased libido and impotence)
amenorrhoea and hypothyroidism may occur rarely.

The superior mesenteric vein and the splenic vein unite behind the neck of the pancreas to form the portal vein.
The portal trunk divides into 2 hepatic lobar veins.
The right branch drains the cystic vein, and the left branch receives the umbilical and paraumbilical veins that enlarge
to form umbilical varices in portal hypertension.

Causes of portal hypertension can be divided into:

PREhepatic
portal vein thrombosis
congenital atresia
INTRAhepatic
liver cirrhosis
hepatic fibrosis
Wilson's disease
hemochromatosis
congenital fibrosis
POSThepatic (occurs at any level between liver and right heart)
hepatic vein thrombosis
inferior vena cava thrombosis
inferior vena cava congenital malformation
constrictive pericarditis

https://www.mcqbank.co.uk/note_manage_print.php Page 18 of 55
06/01/2017, 17)40

Riboflavin or vitamin B2 is the central component of the cofactors FAD (flavin adenine dinucleotide) and FMN (flavin
mononucleotide) and is required for the metabolism of fats, ketone bodies, carbohydrates, and proteins.

Milk, cheese, leafy green vegetables, liver, kidneys, legumes such as mature soybeans, yeast, mushrooms and
almonds are good sources of vitamin B2.
Riboflavin deficiency results in cracked and red lips, inflammation of the lining of mouth and tongue, mouth ulcers, cracks at the
corners of the mouth (angular cheilitis), and a sore throat.

A deficiency may also cause dry and scaling skin, fluid in the mucous membranes, and iron-deficiency anaemia.
The eyes may also become bloodshot, itchy, watery and sensitive to bright light.
Riboflavin deficiency is also classically associated with the oral-ocular-genital syndrome.
Angular cheilitis, photophobia, and scrotal dermatitis are the often quoted signs.

Vitamin Effects of Deficiency


Night blindness
Vit A (retinol)
Dry skin
Beriberi
Vit B1 (thiamine)
Wernicke-Korsakoff
Vit B2 (riboflavin) Angular stomatitis
Vit B3 (niacin) Pellagra
Dermatitis
Enteritis
Vit B5 (panthothenate)
Alopecia
Adrenal insufficiency
Convulsions
Vit B6 (pyridoxine)
Hyperirritability
Dermatitis
Biotin
Enteritis
Macrocytic, megaloblastic anaemia
Folic acid
Sprue
Macrocytic, megaloblastic anaemia
Glossitis
Neurological symptoms
Vitamin B12 (cobalamin)
subacute combined cord degeneration
optic neuropathy
paraesthesia

Vitamin C (ascorbic acid) Scurvy


Rickets in chidren
Vitamin D Osteomalacia in adults
Hypocalcaemic tetany

https://www.mcqbank.co.uk/note_manage_print.php Page 19 of 55
06/01/2017, 17)40

Vitamin E Increased fragility of erythrocytes


Vitamin K Neonatal haemorrhage

Acute mesenteric ischaemia (AMI) includes acute mesenteric arterial embolus and thrombus, mesenteric venous
thrombus, and nonocclusive mesenteric ischemia (NOMI).

They all have the features of impaired blood supply to the intestine, bacterial translocation (the passage of
intestinal bacteria to normally sterile tissue), and systemic inflammatory response.
The overall prevalence is 0.1% of all hospital admissions
Usually affects the over 50’s, although younger people with risk factors can be affected.
Risk factors include:
atrial fibrillation
mitral stenosis post MI
valvular endocarditis
arterial catheterisation
aortic aneurysm or dissection
decreased cardiac output
dehydration
hypotension
hypercoagulability disorders
intra-abdominal infection
Classically the patients complains of acute severe abdominal pain (usually central), but abdominal signs are
lacking (pain out of proportion to examination).
Can lead to hypovolaemic shock.
Bowel sounds are usually absent.
Blood tests may show a raised white cell count and the presence of metabolic acidosis.
Early on the AXR shows a “gasless” abdomen.
Angiography is the gold standard and shows arterial blockage due to emboli or thrombus.
However, many diagnoses are made at laparotomy with the finding of necrotic bowel.

https://www.mcqbank.co.uk/note_manage_print.php Page 20 of 55
06/01/2017, 17)40

The correct answer is colonoscopy and multiple biopsies. Patients with longstanding extensive ulcerative colitis for at
least 10 years' duration are at increased risk of colon cancer. Appropriate surveillance involves colonoscopy (every 1-5
years) with multiple biopsies at regular intervals, even of normal appearing mucosa, to check for dysplasia.

None of the other choices allows sampling of the entire colonic mucosa for histologic examination for the precancerous
lesions of low- or high-grade dysplasia.

Primary biliary cirrhosis is an autoimmune disease of the liver with destruction of the bile canaliculi within the
liver.

Cholestasis occurs leading to scarring, fibrosis and cirrhosis.


May be associated with osteoporosis & sicca syndrome.
It is commoner in women and most occur between 30 and 65 years of age.
Symptoms include fatigue, pruritus, steatorrhoea and jaundice.
Examination findings may include complications of cirrhosis and portal hypertension, xanthelasmata or
xanthomata, skin hyperpigmentation, hepatomegaly (70%), splenomegaly (35%).
Diagnosis should be suspected in a middle-aged woman who presents with fatigue, itching, jaundice or
right upper quadrant discomfort.
The hallmark of this disease is the presence of antimitochondrial antibodies which can be found in 90-95% of
patients with primary biliary cirrhosis, and they have a specificity of 98% for this disease.
The goals of treatment are to slow the progression rate of the disease and to alleviate the symptoms (e.g., pruritus,
osteoporosis, sicca syndrome).
Liver transplantation appears to be the only life-saving procedure.

This patient has symptoms of congestive heart failure and possible atrial fibrillation, as demonstrated by her irregularly
irregular heartbeat. She is at high risk for the development of an embolic occlusion of the superior mesenteric artery.
These patients will present with severe pain out of proportion to their objective physical findings.
Diverticulitis (option A) may present with severe abdominal pain but is generally lower abdominal and is often
localized in the left lower quadrant, the site of sigmoid diverticulitis. Patients will often give a history of chronic
crampy, postprandial pain in the left lower quadrant.
Ischaemic colitis (option B) will usually present as diarrhoea, often bloody, in elderly patients with known
atherosclerotic heart disease.
Although pancreatitis (option D) may develop abruptly, particularly with gallstone pancreatitis, the symptoms are
usually localized to the epigastric region, with radiation to the back and associated nausea and vomiting.
A small bowel obstruction (option E) is unlikely in the absence of prior abdominal surgery, and associated adhesions
and will generally present with abdominal distension in association with high-pitched hyperactive bowel sounds, as well
as nausea and vomiting.

Acute mesenteric ischaemia (AMI) includes acute mesenteric arterial embolus and thrombus, mesenteric venous
thrombus, and nonocclusive mesenteric ischemia (NOMI).

https://www.mcqbank.co.uk/note_manage_print.php Page 21 of 55
06/01/2017, 17)40

They all have the features of impaired blood supply to the intestine, bacterial translocation (the passage of
intestinal bacteria to normally sterile tissue), and systemic inflammatory response.
The overall prevalence is 0.1% of all hospital admissions
Usually affects the over 50’s, although younger people with risk factors can be affected.
Risk factors include:
atrial fibrillation
mitral stenosis post MI
valvular endocarditis
arterial catheterisation
aortic aneurysm or dissection
decreased cardiac output
dehydration
hypotension
hypercoagulability disorders
intra-abdominal infection
Classically the patients complains of acute severe abdominal pain (usually central), but abdominal signs are
lacking (pain out of proportion to examination).
Can lead to hypovolaemic shock.
Bowel sounds are usually absent.
Blood tests may show a raised white cell count and the presence of metabolic acidosis.
Early on the AXR shows a “gasless” abdomen.
Angiography is the gold standard and shows arterial blockage due to emboli or thrombus.
However, many diagnoses are made at laparotomy with the finding of necrotic bowel.

Q1 - G

Colorectal carcinoma

3rd most commonly diagnosed cancer in UK (after Breast & lung).


Colon cancer is equally common in men and women, but rectal cancer is more common in men.
Risks factors for colorectal carcinoma include:
Age > 40 yr
Lifestyle - Dietary factors
Genetic factors:
familial adenomatous polyposis

https://www.mcqbank.co.uk/note_manage_print.php Page 22 of 55
06/01/2017, 17)40

Gardner's and Turcot's syndromes


familial colorectal cancer syndrome (Lynch I)
hereditary adenocarcinomatosis syndrome (Lynch II)
family history of colorectal carcinoma
Colorectal disease:
inflammatory bowel disease
benign colorectal polyps
personal history of colorectal cancer
pelvic irradiation
Other:
Hormone replacement therapy - reduces the chance of colorectal cancer
The distribution of colorectal cancer is:
45% rectum
25% sigmoid colon
15% caecum and ascending colon
10% transverse colon
5% descending colon
The presentation depends on the site of the cancer:
Right colon cancers
weight loss, anaemia, occult bleeding, mass in right iliac fossa
disease more likely to be advanced at presentation.
Left colon cancers
colicky pain, rectal bleeding, bowel obstruction, tenesmus, mass in left iliac fossa, early change
in bowel habit
less advanced disease at presentation.
Investigations
Colonoscopy is the gold standard for diagnosis of colorectal cancer.
Computed tomographic (CT) colonography is an effective, safe method for examining the colon and
rectum to detect abnormalities such as polyps and cancer.
Liver ultrasound, (occasionally intrarectal ultrasound) and CT or magnetic resonance imaging (MRI)
are useful in staging. MRI is more specific than CT in showing liver metastases.
2015 Cancer referral guidelines (see https://www.nice.org.uk/guidance/ng12):
REFER 2ww for colorectal cancer if:
≥40 with unexplained WEIGHT LOSS and ABDOMINAL PAIN.
≥50 with unexplained RECTAL BLEEDING.
≥60 with:
IRON-DEFICIENCY ANAEMIA.
CHANGE IN BOWEL HABIT.
+ve FAECAL OCCULT BLOOD.
CONSIDER 2ww referral for colorectal cancer in:
People with a rectal or abdominal mass.
Adults <50 with rectal bleeding AND any of the following unexplained symptoms:
Abdominal pain.
Change in bowel habit.
Weight loss.
Iron deficiency anaemia.
Offer faecal occult blood testing in adults without rectal bleeding who are:
≥50 with unexplained:
Abdominal pain.
Weight loss.
<60 with:
Changes in bowel habit.
Iron deficiency anaemia.
≥60 with:
Anaemia even in the absence of iron deficiency.
Staging
The Dukes' staging classification is now gradually being replaced by the tumour/node/metastases (TNM)

https://www.mcqbank.co.uk/note_manage_print.php Page 23 of 55
06/01/2017, 17)40

classification.
Management
Surgery remains the definitive treatment for apparently localised colorectal cancer.
Both radiotherapy and chemotherapy can improve survival rates after potentially curative surgery, and
chemotherapy prolongs overall survival in patients with advanced disease.

Q2 - G

This patient's constipation is most likely secondary to hypercalcaemia from her metastatic breast tumour.

The common causes of hypercalcaemia include:

1) Malignancy

Accounts for 50% of cases


Mechanism:
Metastases with osteolytic deposits e.g. breast carcinoma
Osteoclast activating factors e.g. multiple myeloma
Ectopic parathyroid hormone like peptide e.g. hypernephroma, ovarian tumour, bronchial carcinoma
Frequently accompanied by hypoalbuminaemia

(2) Primary hyperparathyroidism

Accounts for 20% of cases


More important in younger patients
Presentation can be remembered by the mnemonic "Bones, Stones, Groans & Psychic moans"
Abdominal pain
Vomiting
Constipation
Polyuria
Polydipsia
Depression
Anorexia
Weight loss
Tiredness
Weakness
Hypertension
Confusion
Pyrexia
Renal stones
Renal failure
Corneal calcification
Cardiac arrest
ECG (QT interval ↓)

Q3 - D

Constipation is infrequent or difficult evacuation of faeces.

Causes include:
organic obstruction
painful anal conditions
adynamic bowel
drugs, habit and diet

https://www.mcqbank.co.uk/note_manage_print.php Page 24 of 55
06/01/2017, 17)40

metabolic imbalance
Hypothyroidism
Hypercalcaemia
Hypokalaemia
Porphyria
Lead poisoning​
Clinical features include infrequent, incomplete evacuation of stools (generally this is taken to be less than
twice a week
however, normal bowel habit can vary tremendously and what is considered constipation for some is
diarrhoea for others!
More significant is the change in bowel habit relative to the patient's normal bowel habit,
Other symptoms may include anorexia and vague abdominal discomfort, diarrhoea (constipation may
cause overflow or spurious diarrhoea, especially in the elderly, when faecal fluid intermittently escapes past
an impacted faecal mass), acute abdominal pain (usually in children), or features of intestinal
obstruction (usually in the elderly).
On examination, there may be mild abdominal tenderness and a faecally-loaded colon on the left side.
There is usually a mass of faeces felt on rectal examination.
In elderly patients, faeces may be impacted higher up and so an empty rectum does not exclude
constipation.
In children, constipation is usually diagnosed based on the history and the examination.
If chronic constipation occurs, then more extensive investigation for conditions such as Hirschsprung's
disease is warranted.
Principles of management in adults:
Treat the cause.
Mobilise the patient.
Consider drugs only if above measures fail.
Try to use drugs for short durations only.

This patient's presentation is consistent with acute hepatitis. Although alcoholic hepatitis may be indistinguishable
from other forms of acute hepatitis, an alcoholic aetiology is favoured by the raised AST (>2x) compared with ALT.

All the remaining laboratory findings, e.g., neutropaenia, hypoalbuminaemia and prolonged PT, may be present in
many other forms of acute hepatitis, including viral hepatitis (option E).
Evidence of malnourishment is also consistent with alcoholism.
Macrocytic anaemia is a frequent coexisting finding due to vitamin B12 and folic acid deficiency.

Acute pancreatitis (option A) most frequently develops in patients with a history of alcoholism or cholelithiasis.

It presents with a dramatic picture of extremely severe, deep abdominal pain often radiating to the back. The
patient is restless and diaphoretic.
Serum levels of amylase and lipase are usually markedly elevated. Mild elevations of these enzymes are also
often seen in alcoholic hepatitis.

Cholecystitis (option C) is usually secondary to a stone impacted in the cystic duct, resulting in distention of the
gallbladder and colicky pain.

Fever and mild jaundice may be present, but usually AST and ALT are normal or slightly elevated.

Duodenal peptic ulcer (option D) is accompanied by epigastric pain or discomfort.

If perforation occurs, the pain begins suddenly and is associated with abdominal guarding.

https://www.mcqbank.co.uk/note_manage_print.php Page 25 of 55
06/01/2017, 17)40

Jaundice, laboratory evidence of liver damage, and ascites are absent.

Endoscopic retrograde cholangiopancreatography (ERCP) is used both in the diagnosis and treatment of many
pancreatic and biliary diseases.

It was first used in the 1970s at which point its main use was in diagnosis - in contrast to nowadays when its use is
mostly as a therapeutic tool.
ERCP has a role to play in the management of choledocholithiasis especially in the setting of obstructive
jaundice.
Patients are considered too high risk for cholecystectomy if they have had recent cholangitis or recent
acute pancreatitis or in the presence of jaundice, abnormal LFT's (ALP more than twice normal) or a
dilated CBD >10 mm.

Haemorrhoids are enlarged congested mucosal subepithelial vascular cushions found in the 3, 7 & 11 o’clock
positions that contribute to anal continence.

They are the commonest cause of haematochezia.


Factors contributing to haemorrhoids include a
low fibre diet
increased anal sphincter tone
obstruction of venous flow during pregnancy
portal hypertension
Haemorrhoids may be either
internal (origin above the dentate line)
external (origin below the dentate line)
can be classified into four grades
I - do not prolapse out of the anal canal
II - prolapse on defecation but reduce spontaneously
III - require manual reduction
IV - cannot be reduced
Haemorrhoids often only produce intermittent symptoms
Symptomatic episodes are often precipitated by constipation.
These episodes usually last from a few days to a few weeks.
Often they are completely asymptomatic between episodes.
Clinical features of piles include
bleeding after defecation (which may just stain the toilet paper or streak the faeces or if copious it
may splash around the lavatory pan)
faecal soiling
mucous discharge
pruritus ani
occasionally pain (thrombosis may cause acute severe anal pain occasionally necessitating hospital
admission)
rectal mass (grades II-IV)
Note haemorrhoids are often not palpable on digital examination because they empty with pressure from the
examining finger.
Proctoscopy is necessary to diagnose first or second degree piles where the piles are seen bulging into the lumen

https://www.mcqbank.co.uk/note_manage_print.php Page 26 of 55
06/01/2017, 17)40

as the proctoscope is withdrawn


A sigmoidoscopy is indicated if there is a history of bleeding or there are symptoms that are suspicious of
malignancy
Treatment includes
Conservative measures such as
improving diet
ensuring the perineum is dried and washed after defecation
digital replacement of prolapsed haemorrhoids can lead to symptomatic relief
local anaesthetic creams and ointments
glyceryl trinitrate 0.2-0.3% (unlicensed)
Outpatient interventions appropriate for grade I or II haemorrhoids include
sclerotherapy
rubber band ligation
photocoagulation
cryotherapy
botulinum toxin injection may help reduce spasm of the internal anal sphincter
Surgical options include
Lord's stretch procedure & haemorrhoidectomy

Abdominal plain film is the best choice here. This will show clearly a dilated (>6 cm) colon and sometimes loss of
colonic haustrations and possible "thumbprinting":

Barium enema and colonoscopy are best avoided due to the high risk of perforation. Erect CXR is best for
demonstrating free gas in the peritoneum following perforation. Ultrasound of the abdomen is a potential diagnostic
test but many gastroenterologists would prefer the abdominal film. A CT abdomen would be the next investigation as it
may identify a local or contained perforation or may be useful in equivocal cases or to exclude abscess as the cause of
the toxicity.

Toxic megacolon is an acute toxic colitis with dilatation of the colon which can be either total or segmental. Note,
patients may develop toxicity without megacolon.

Jalan's diagnostic criteria include:

1. Radiographic evidence of colonic dilatation (>6cm)

2. Any 3 of the following:

https://www.mcqbank.co.uk/note_manage_print.php Page 27 of 55
06/01/2017, 17)40

fever (>38.5°C)
tachycardia (>120 beats/min)
leukocytosis (>10.5x103/µL)
anaemia

3. Any 1 of the following:

dehydration
altered mental status
electrolyte abnormality
hypotension

Acute cholecystitis follows impaction of a stone in the cystic duct, which may cause continuous epigastric or right
upper quadrant pain, vomiting, fever, local peritonism, or a gallbladder mass.

The main difference from biliary colic is the inflammatory component (local peritonism, fever, raised white cell
count).
If the stone moves to the common bile duct jaundice may occur.
Repeated attacks of acute cholecystitis lead to chronic cholecystitis, in which the walls of the gallbladder become
thickened and scarred, and the gallbladder becomes shrivelled.
Murphy's sign is performed by asking the patient to breathe out and then gently placing the hand below the
costal margin on the right side at the mid-clavicular line (the approximate location of the gallbladder).
The patient is then asked to breathe in.
During inspiration, the abdominal contents are pushed downward and the gallbladder will meet the
examiner’s hand.
If the gallbladder is tender this will cause the patient to stop inspiring due to pain.
This is considered positive but a positive test also requires no pain on performing the manoeuvre on
the patient's left hand side.
Typically, it is positive in cholecystitis, but negative in choledocholithiasis and ascending cholangitis.

Ruptured Abdominal Aortic Aneurysm has a mortality of 40% if treated and 100% if untreated.

The patient is usually over 50 years old.


Presents with
intermittent or continuous abdominal pain that can radiate to the back, iliac fossae or groins (& hence be
confused with renal colic)
shock
collapse
sometimes an expansile abdominal mass can be palpated.

https://www.mcqbank.co.uk/note_manage_print.php Page 28 of 55
06/01/2017, 17)40

Ultrasonography is the most diagnostic test of choice for diagnosis of biliary colic and acute cholecystitis.

It is 90-95% sensitive for cholecystitis and 78-80% specific. For simple cholelithiasis, it is 98% sensitive and
specific.
Ultrasound can reveal gallstones or sludge, gallbladder wall thickening, gallbladder distention, presence of
pericholecystic fluid from perforation or air in the gallbladder wall (indicating gangrenous cholecystitis).
In addition a sonographic Murphy sign is pain when the probe is pushed directly on the gallbladder (not
related to breathing).
Gallbladder with a row of small gallstones causing an acoustic shadow:

Loperamide is contraindicated in the treatment of active ulcerative colitis.

Ulcerative colitis (UC) is an idiopathic chronic inflammatory disease of the colon that follows a course of relapse and
remission.

Affects 1 in 1000
The genetic component is not as strong in ulcerative colitis as it is in Crohn's disease
The cardinal symptom is bloody diarrhoea.
Associated symptoms include colicky abdominal pain, urgency, or tenesmus
Approximately 4% of patients will have extraintestinal disease which may include:
Related to the activity of colitis:
Erythema nodosum
Aphthous ulcers
Episcleritis
Acute arthropathy affecting the large joints (e.g. wrists, hips, knees)
Usually related to activity of colitis:
Pyoderma gangrenosum
Anterior uveitis
Not related to activity of colitis:
Sacroiliitis
Ankylosing spondylitis
Primary sclerosing cholangitis
The diagnosis should be made on the basis of clinical suspicion supported by appropriate macroscopic findings on
sigmoidoscopy or colonoscopy, typical histological findings on biopsy, and negative stool examinations for
infectious agents.

https://www.mcqbank.co.uk/note_manage_print.php Page 29 of 55
06/01/2017, 17)40

Management
Topical management is appropriate for some patients with active disease.
This is usually the case for those with proctitis and often the case if the disease extends into the
sigmoid.
For those with more extensive disease, oral or parenteral therapy are the mainstays of treatment, although
some of these patients may get additional benefit from topical therapy.
Drug treatments
Aminosalicylates
e.g. Mesalazine or 5-aminosalicylic acid (5-ASA)
Corticosteroids
Used to induce remission in relapses of ulcerative colitis.
They have no role in maintenance therapy.
Azathioprine
Patients are intolerant to corticosteroids
Patients need two or more corticosteroid courses in a calendar year
Cyclosporin
Used for effective salvage therapy for patients with severe refractory colitis and has a rapid
onset of action.
Infliximab
effective in inducing clinical remission in patients with moderate to severe ulcerative colitis
whose disease is refractory to conventional treatment using corticosteroids and/or
immunosuppressive agents.

Q1 - B

Femoral hernias occur below the inguinal ligament, where abdominal contents pass through the femoral canal.

In 2005/6 there were 4,583 femoral hernia repairs in the NHS (c.f. 79,760 for inguinal hernia) in England & of
these, 49% were emergency repairs and 48% waiting list repairs.
Femoral hernia is 4 times commoner in women but in women, inguinal hernia is still more common than
femoral hernia.
The incidence is highest in middle-aged and elderly women, especially if parous.
It presents as a lump in the groin, lateral and inferior to the pubic tubercle (inguinal hernias are superior and
medial) but a large hernia may bulge over the inguinal ligament and make differential diagnosis difficult.
The lump is worse on coughing or straining and reduces in size or disappears when relaxed or supine.
The risk of strangulation in a femoral hernia is much higher than for an inguinal hernia and only 50% of patients
are aware of the hernia before strangulation.
A strangulated hernia presents as a red tender, tense irreducible lump in conjunction with signs of bowel
obstruction.

Q2 - E

A Spigelian hernia (or lateral ventral hernia) is a rare hernia through the spigelian fascia, which is the aponeurotic
layer between the rectus abdominis muscle medially, and the semilunar line laterally.

These hernias almost always develop at or below the linea arcuata, probably because of the lack of posterior
rectus sheath.
These are generally interparietal hernias, meaning that they do not lie below the subcutaneous fat but
penetrate between the muscles of the abdominal wall.
Therefore, there is often no notable swelling.
Spigelian hernias are usually small and therefore risk of strangulation is high.
Most occur on the right side and most develop around age 50.

https://www.mcqbank.co.uk/note_manage_print.php Page 30 of 55
06/01/2017, 17)40

Q3 - H

The iatrogenic incisional hernia develops as a late complication of about 10% of abdominal surgery.

The hernia may be apparent within the first year post-operatively, but may not develop for up to 5 years.
Usually, the incisional hernia presents as a bulge in the abdominal wall near a previous wound.
The condition is often asymptomatic but occasionally, a narrow-necked hernia occurs and presents with
pain or strangulation.
Once developed, it tends to enlarge progressively and may become a nuisance cosmetically or for dressing.
Treatment is by surgical repair.
Even after repair, recurrence rates approach 20-45%.

This patient is more likely to have a peptic ulcer secondary to high dose steroid.

Crohn's disease is a disorder of unknown aetiology that is characterised pathologically by involvement of all bowel
wall layers in a chronic inflammatory process with non-caseating granulomas.

The granulomatous inflammation most frequently affects the terminal ileum but it may affect any part of the
gastrointestinal tract and frequently affected areas are in discontinuity.
Possible local complications of include:
intestinal obstruction
haemorrhage
perforation with abscess
stricture formation
perianal abscess
fistulae to the bowel, bladder, vagina
increased risk of malignancy but less than that of ulcerative colitis
Common general complications include:
weight loss
anaemia
arthritis of the large joints
erythema nodosum
ocular problems (conjunctivitis, episcleritis, iritis)
sacroiliitis (this is unrelated to HLA B27)
gall stones, especially of the cholesterol type

Carcinoma of the oesophagus is the ninth most common malignant tumour in the UK.

The incidence of oesophageal carcinoma varies considerably with geographical location with high rates in China
and Iran, where it has been directly linked to the preservation of food using nitrosamines.
Oesophageal carcinoma usually affects the elderly and can be either squamous cell carcinoma or
adenocarcinoma.
Risk factors
Tobacco and alcohol are strong risk factors
Barrett's oesophagus
Chronic inflammation and stasis from any cause increase the risk, e.g. strictures due to caustic injury or

https://www.mcqbank.co.uk/note_manage_print.php Page 31 of 55
06/01/2017, 17)40

achalasia.
Tylosis and Plummer-Vinson syndrome
Obesity - increases the risk of gastro-oesophageal reflux disease in turn increasing the risk of Barrett's
oesophagus
Oesophageal carcinoma presents with progressive mechanical dysphagia.
Patients usually have rapidly progressive dysphagia initially with solids then with liquids
Weight loss (> 10 kg)
Loss of appetite
Investigations
Urgent endoscopy - with brushings and biopsy of any lesion seen.
Primary treatment modalities include surgery alone or with chemotherapy or radiation therapy.

Diarrhoea with flushing should prompt you to search for endocrine causes.

A glucagonoma is a rare neuroendocrine tumor with nearly exclusive pancreatic localization.

There is overproduction of glucagon which increases blood glucose levels through gluconeogenesis and
lipolysis.
Presents with necrolytic migratory erythema (NME) in 80% which is characterized by an erythematous and
swollen area of skin.
Diabetes or glucose intolerance occurs in 80-90% and weight loss is present in most patients.
DVT occurs in 40% and diarrhoea & abdominal pains in 20%.
Depression may also be present hence the 4Ds (diarrhoea, diabetes, DVT, depression).

A gluten-free diet is recommended amongst other things in the treatment of coeliac disease and wheat allergy.

It is a diet completely free of ingredients derived from gluten-containing cereals:

wheat (including kamut and spelt)


barley
rye
triticale
as well as the use of gluten as a food additive in the form of a flavoring, stabilizing or thickening agent.

Oats may or may not be included in this list (controversial).

Chronic cholecystitis manifests with non-specific symptoms such as nausea, vague abdominal pain, belching, and
diarrhoea.

Biliary colic occurs when there is obstruction of the cystic duct by a gallstone.

It is described as colic because it occurs intermittently but the pain, when it occurs, is actually continuous - thus it

https://www.mcqbank.co.uk/note_manage_print.php Page 32 of 55
06/01/2017, 17)40

is not a true colic.


It usually begins suddenly (often occurring postprandially) and subsides gradually, lasting from a few minutes to
several hours and is felt in the right upper quadrant but epigastric and left abdominal pain are common, and
some patients may experience chest pain.
The pain may radiate around the costal margin into the back or may be referred to the region of the
scapula.
There may be nausea and often a bout of vomiting signifies the end of an attack.
The frequency of attacks is very variable ranging from almost continuous to many years apart.
There is no fever but the patient may be mildly tachycardic because of fright from the intensity of the pain.
Often, there is local tenderness due to gallbladder distension and this may preclude deep palpation.
Other symptoms associated with gallstone disease may be present, including intolerance of fatty foods,
dyspepsia, and flatulence.
There may be transient increases in serum bilirubin and alkaline phosphatase.
Ultrasound may reveal stones in the cystic duct or the gallbladder.
First line treatment is a NSAID such as IM diclofenac.
Severe attacks may lead to hospital admission.
If the patient is fit for surgery then cholecystectomy is indicated.
This is usually done electively.
If the patient is unfit for surgery then dissolution therapy with ursodeoxycholic acid may be indicated for patients
with radiolucent stones less than 1.5cm in diameter and with a functioning gallbladder verified on oral
cystography.
There is complete dissolution in about 30% of cases at 12 months.

Risk factors for gastric carcinoma:

1. Increasing age
90% of gastric cancers occur in those aged >55 years
2. More common in men than women
3. Poor socio-economic status
4. Helicobacter pylori
there is a 2.5x increased risk of gastric cancer in infected individuals.
5. Diet:
diets containing low levels of fresh fruit and vegetable consumption increase the risk of gastric cancer.
high levels of salt and preserved foods may also increase the risk.
6. Smoking
7. Atrophic gastritis, pernicious anaemia, post-gastrectomy, Ménétrier's disease
8. Familial risk:
There is a 2-3x increased risk in first-degree relatives
There is a link between E-cadherin gene mutations and some familial gastric cancers.
9. Blood group A (relative risk is 1.2)
10. Hypogammaglobulinaemia

Spontaneous bacterial peritonitis (SBP) is an acute bacterial infection of ascitic fluid.

Presents as fever and abdominal discomfort.


Other presentations include worsening or unexplained encephalopathy or ascites that does not improve

https://www.mcqbank.co.uk/note_manage_print.php Page 33 of 55
06/01/2017, 17)40

following administration of diuretic medication.


All patients suspected of having SBP must undergo diagnostic paracentesis or peritoneal fluid analysis.
Empirical antibiotics (usually an intravenous third-generation cephalosporin) is commenced if the ascitic fluid
polymorphonuclear neutrophil counts greater than 250 cells/µL.

Colorectal carcinoma

3rd most commonly diagnosed cancer in UK (after Breast & lung).


Colon cancer is equally common in men and women, but rectal cancer is more common in men.
Risks factors for colorectal carcinoma include:
Age > 40 yr
Lifestyle - Dietary factors
Genetic factors:
familial adenomatous polyposis
Gardner's and Turcot's syndromes
familial colorectal cancer syndrome (Lynch I)
hereditary adenocarcinomatosis syndrome (Lynch II)
family history of colorectal carcinoma
Colorectal disease:
inflammatory bowel disease
benign colorectal polyps
personal history of colorectal cancer
pelvic irradiation
Other:
Hormone replacement therapy - reduces the chance of colorectal cancer
The distribution of colorectal cancer is:
45% rectum
25% sigmoid colon
15% caecum and ascending colon
10% transverse colon
5% descending colon
The presentation depends on the site of the cancer:
Right colon cancers
weight loss, anaemia, occult bleeding, mass in right iliac fossa
disease more likely to be advanced at presentation.
Left colon cancers
colicky pain, rectal bleeding, bowel obstruction, tenesmus, mass in left iliac fossa, early change
in bowel habit
less advanced disease at presentation.
Investigations
Colonoscopy is the gold standard for diagnosis of colorectal cancer.
Computed tomographic (CT) colonography is an effective, safe method for examining the colon and
rectum to detect abnormalities such as polyps and cancer.
Liver ultrasound, (occasionally intrarectal ultrasound) and CT or magnetic resonance imaging (MRI)
are useful in staging. MRI is more specific than CT in showing liver metastases.
2015 Cancer referral guidelines (see https://www.nice.org.uk/guidance/ng12):
REFER 2ww for colorectal cancer if:
≥40 with unexplained WEIGHT LOSS and ABDOMINAL PAIN.
≥50 with unexplained RECTAL BLEEDING.
≥60 with:
IRON-DEFICIENCY ANAEMIA.

https://www.mcqbank.co.uk/note_manage_print.php Page 34 of 55
06/01/2017, 17)40

CHANGE IN BOWEL HABIT.


+ve FAECAL OCCULT BLOOD.
CONSIDER 2ww referral for colorectal cancer in:
People with a rectal or abdominal mass.
Adults <50 with rectal bleeding AND any of the following unexplained symptoms:
Abdominal pain.
Change in bowel habit.
Weight loss.
Iron deficiency anaemia.
Offer faecal occult blood testing in adults without rectal bleeding who are:
≥50 with unexplained:
Abdominal pain.
Weight loss.
<60 with:
Changes in bowel habit.
Iron deficiency anaemia.
≥60 with:
Anaemia even in the absence of iron deficiency.
Staging
The Dukes' staging classification is now gradually being replaced by the tumour/node/metastases (TNM)
classification.
Management
Surgery remains the definitive treatment for apparently localised colorectal cancer.
Both radiotherapy and chemotherapy can improve survival rates after potentially curative surgery, and
chemotherapy prolongs overall survival in patients with advanced disease.

Q1 - D

Note NICE Guidelines CG86 (2009) - Coeliac disease: Recognition and assessment of coeliac disease, prefers IgA
tissue transglutaminase antibody for the diagnosis of coeliac disease, but some laboratories will routinely use IgA
endomysial antibody. These are both more accurate than anti-gliadin antibody. Anti-gliadin antibodies are anti-food
protein antibodies and as such, they are not indicative of any autoimmune reactions. They appear only if the patient has
been eating gluten, but they are not linked to any detectable adverse reaction to gluten. In other words, they appear in
individuals who eat gluten as a response to it touching the gut, but do not necessarily correlate with any clinical
pathology.

Coeliac disease (CD) is an autoimmune gluten-sensitive enteropathy occurring in genetically predisposed


individuals.

The disease primarily affects the small intestine, where it progressively leads to flattening of the small intestinal
mucosa.
Three cereals contain gluten and are toxic for patients with celiac disease: wheat, rye, and barley.
CD is commoner in females and classically presents in children aged 9-18 months (but it can occur at any stage
in life and in adults it is usually precipitated by an infectious diarrhoeal episode or other intestinal disease).
The typical presentation occurs at various times after the introduction of foods that contain gluten.
Infants and young children typically present with chronic diarrhoea, anorexia, abdominal distension,
abdominal pain, poor weight gain or weight loss, and vomiting.
Severe malnutrition can occur if the diagnosis is delayed.
Behavioural changes are common and include irritability and an introverted attitude.
Rarely, severely affected infants present with a coeliac crisis, which is characterized by explosive watery
diarrhoea, marked abdominal distension, dehydration, hypotension, and lethargy, often with profound
electrolyte abnormalities, including severe hypokalemia.

https://www.mcqbank.co.uk/note_manage_print.php Page 35 of 55
06/01/2017, 17)40

Q2 – F

Irritable bowel syndrome (IBS) is a chronic, relapsing disorder characterised by abdominal discomfort, which may be
associated with defecation and/or accompanied by a change in bowel habit.

IBS is the most common functional GI disorder - there is no structural lesion, but it may be explained by
abnormal smooth muscle activity ± visceral hypersensitivity.
IBS may present at any age (peak prevalence in 30s and 40s - female predominance is most obvious in the 3rd
decade and declines afterwards).
NICE (http://www.nice.org.uk/guidance/CG61) states that a diagnosis of IBS should be considered only if the
person has abdominal pain or discomfort that is either relieved by defecation or associated with altered
bowel frequency or stool form.
This should be accompanied by at least two of the following four symptoms:
altered stool passage (straining, urgency, incomplete evacuation)
abdominal bloating (more common in women than men), distension, tension or hardness
symptoms made worse by eating
passage of mucus
Other features such as lethargy, nausea, backache and bladder symptoms are common in people with IBS, and
may be used to support the diagnosis.
Investigations should be limited to those that are likely to exclude serious alternative diagnoses and when
negative support a positive diagnosis of IBS.
The most important differential diagnoses are coeliac disease, colorectal carcinoma and colitis.
Management:
General dietary advice
Have regular meals - i.e. avoid long gaps between meals and don't rush them.
Drink plenty of fluids (at least 8 cups per day) but restrict tea/coffee to 3 cups or so per day.
Reduce intake of alcohol and fizzy drinks.
Consider limiting high-fibre foods (e.g. wholemeal flour or bran), and resistant starches (often in
processed or recooked foods, and fresh fruits - limit to 3 portions per day).
For diarrhoea - avoid sorbitol.
For wind - consider increasing oats and linseeds (one tablespoon/day).
Food intolerance is common with IBS although true allergies are rare - suggest omitting any known food triggers,
but a formal exclusion diet needs the support of a dietician.
Drug therapy is directed towards symptomatic relief:
Antispasmodics especially mebeverine through their anti-muscarinic actions may relieve pain by
moderating smooth-muscle contractions.
Peppermint oil before meals may be of benefit with colonic spasm and symptoms of bloating
NICE suggest:
Laxatives should be considered for the treatment of constipation in people with IBS, but people should be
discouraged from taking lactulose.
Loperamide should be the first choice of antimotility agent for diarrhoea in people with IBS.
Tricyclic antidepressants (TCAs) as second-line treatment for people with IBS if laxatives, loperamide or
antispasmodics have not helped.
Selective serotonin reuptake inhibitors (SSRIs) should be considered for people with IBS only if TCAs
have been shown to be ineffective.
More than 50% will continue to have symptoms after 5 years.

Q3 - B

Cystic fibrosis is the most common potentially lethal autosomal recessive disorder in Caucasian populations
(prevalence is 1 in 2,500 with calculated carrier frequency of 1 in 25), characterised by chronic suppurative lung
disease and chronic exocrine pancreatic insufficiency.

It is caused by mutations in the cystic fibrosis transmembrane conductance regulator (CFTR) gene, on
chromosome 7.

https://www.mcqbank.co.uk/note_manage_print.php Page 36 of 55
06/01/2017, 17)40

CFTR is an ATP-responsive chloride channel that also affects other cellular activities, such as sodium
transport across the respiratory epithelium, composition of cell surface glycoprotein and antibacterial
defences.
Sweat testing confirms the diagnosis - chloride concentration >60mmol/l with sodium concentration lower than
that of chloride.
The abnormality in CFTR explains the pathology of cystic fibrosis.
High sodium in sweat
Primary secretion of sweat duct is normal, but CFTR does not absorb chloride ions, which remain in
the lumen and prevent sodium absorption.
Pancreatic insufficiency
Production of pancreatic enzymes is normal but defects in ion transport produce relative dehydration
of pancreatic secretions causing their stagnation in the pancreatic ducts.
Biliary disease
Defective ion transfer across bile duct causes reduced movement of water in the lumen so that bile
becomes concentrated causing plugging and local damage.
GI disease
Low volume secretions of increased viscosity
Changes in fluid movement across both small and large intestine
Dehydrated biliary and pancreatic secretions cause intra luminal water deficiency
Respiratory disease
Dehydration of the airway surfaces reduces mucociliary clearance and favours bacterial colonisation
Local bacterial defences are impaired by local salt concentrations
Bacterial adherence is increased by changes in cell surface glycoproteins
Increased bacterial colonisation and reduced clearance
Produces inflammatory lung damage due to an exuberant neutrophilic response involving mediators
such as IL8 and neutrophil elastase.
Reproductive health and fertility
Nearly all males have obstructive azoospermia - normal spermatogenesis but no vas deferens. Early
counselling should be offered about infertility and sperm count. IVF with aspirated sperm has been
used successfully.
Women are generally of normal fertility but need genetic counselling - inherent risk to pregnancy
with severe lung disease.
Offspring of patients with CF will be carriers of cystic fibrosis.

NSAIDs may actually be protective - one theory being that patients on NSAIDs will be more likely to bleed so hence
more likely to present earlier with PR bleeding. Ref - Non-steroidal anti-inflammatory drugs and molecular
carcinogenesis of colorectal carcinomas. Huls G, Koornstra JJ, Kleibeuker JH. Lancet; 2003 Jul 19 ; 362(9379):230-2.

Colorectal carcinoma

3rd most commonly diagnosed cancer in UK (after Breast & lung).


Colon cancer is equally common in men and women, but rectal cancer is more common in men.
Risks factors for colorectal carcinoma include:
Age > 40 yr
Lifestyle - Dietary factors
Genetic factors:
familial adenomatous polyposis
Gardner's and Turcot's syndromes

https://www.mcqbank.co.uk/note_manage_print.php Page 37 of 55
06/01/2017, 17)40

familial colorectal cancer syndrome (Lynch I)


hereditary adenocarcinomatosis syndrome (Lynch II)
family history of colorectal carcinoma
Colorectal disease:
inflammatory bowel disease
benign colorectal polyps
personal history of colorectal cancer
pelvic irradiation
Other:
Hormone replacement therapy - reduces the chance of colorectal cancer
The distribution of colorectal cancer is:
45% rectum
25% sigmoid colon
15% caecum and ascending colon
10% transverse colon
5% descending colon
The presentation depends on the site of the cancer:
Right colon cancers
weight loss, anaemia, occult bleeding, mass in right iliac fossa
disease more likely to be advanced at presentation.
Left colon cancers
colicky pain, rectal bleeding, bowel obstruction, tenesmus, mass in left iliac fossa, early change
in bowel habit
less advanced disease at presentation.
Investigations
Colonoscopy is the gold standard for diagnosis of colorectal cancer.
Computed tomographic (CT) colonography is an effective, safe method for examining the colon and
rectum to detect abnormalities such as polyps and cancer.
Liver ultrasound, (occasionally intrarectal ultrasound) and CT or magnetic resonance imaging (MRI)
are useful in staging. MRI is more specific than CT in showing liver metastases.
2015 Cancer referral guidelines (see https://www.nice.org.uk/guidance/ng12):
REFER 2ww for colorectal cancer if:
≥40 with unexplained WEIGHT LOSS and ABDOMINAL PAIN.
≥50 with unexplained RECTAL BLEEDING.
≥60 with:
IRON-DEFICIENCY ANAEMIA.
CHANGE IN BOWEL HABIT.
+ve FAECAL OCCULT BLOOD.
CONSIDER 2ww referral for colorectal cancer in:
People with a rectal or abdominal mass.
Adults <50 with rectal bleeding AND any of the following unexplained symptoms:
Abdominal pain.
Change in bowel habit.
Weight loss.
Iron deficiency anaemia.
Offer faecal occult blood testing in adults without rectal bleeding who are:
≥50 with unexplained:
Abdominal pain.
Weight loss.
<60 with:
Changes in bowel habit.
Iron deficiency anaemia.
≥60 with:
Anaemia even in the absence of iron deficiency.
Staging
The Dukes' staging classification is now gradually being replaced by the tumour/node/metastases (TNM)
classification.

https://www.mcqbank.co.uk/note_manage_print.php Page 38 of 55
06/01/2017, 17)40

Management
Surgery remains the definitive treatment for apparently localised colorectal cancer.
Both radiotherapy and chemotherapy can improve survival rates after potentially curative surgery, and
chemotherapy prolongs overall survival in patients with advanced disease.

Acute appendicitis is sudden inflammation of the appendix, usually caused by obstruction of the lumen resulting in
invasion of the appendix wall by the gut flora.

Most common cause of an acute abdomen in the UK - about 10% of the population will develop acute
appendicitis.
Presentation
Classically periumbilical pain moves to the RIF.
Anorexia, vomiting, constipation, furred tongue, tachycardia, pyrexia 37.5-38.5°C, lying still, foetor
and flushed appearance may occur.
Rovsing's sign is positive when palpation of the lower left quadrant of a person's abdomen results in more
pain in the right lower quadrant indicating the patient may have appendicitis.
Investigations
Appendicitis is essentially a clinical diagnosis but the following may be useful:
Urinalysis may exclude urinary tract infection.
Pregnancy test to exclude ectopic pregnancy
Bloods - mild leukocytosis with raised inflammatory markers but bloods may be normal
Ultrasound may help in some patients where the diagnosis is doubtful and in the assessment of an
appendix mass or abscess.
Management
All suspected cases should be admitted to hospital
Appendicectomy is the treatment of choice and is increasingly done as a laparoscopic procedure.
Complications
Perforation - higher risk in elderly people and young children
Appendix mass - omentum and small bowel adhere to the appendix
usually presents with a fever and a palpable mass
Appendix abscess can be shown by ultrasound or CT scan
initial treatment is by percutaneous or open drainage
Pelvic or subphrenic abscess
Adhesions in the longer term may cause intestinal obstruction

Q1 - F

Left atrial hypertrophy occurs in mitral stenosis and places external pressure on the oesophagus.

Q2 - A

Achalasia is an idiopathic motility disorder characterized by failure of lower oesophageal sphincter relaxation as well
as loss of peristalsis in the distal oesophagus.

This impairs the ability of the oesophagus to empty contents into the stomach.
Patients usually complain of dysphagia to both solids and liquids.
Dysphagia to liquids, in particular, is a characteristic of achalasia.
Other symptoms of achalasia include regurgitation, night coughing, chest pain, weight loss, and heartburn.

https://www.mcqbank.co.uk/note_manage_print.php Page 39 of 55
06/01/2017, 17)40

Q3 - G

Gastroesophageal reflux disease (GORD) occurs when the amount of gastric juice that refluxes into the
oesophagus exceeds the normal limit and causes symptoms.

Abnormal gastroesophageal reflux is caused by the functional or mechanical problems with the lower
oesophageal sphincter (LOS).
It is equally common in both sexes and prevalence increases in the over 40 year olds.
Predisposing factors:
Certain foods (e.g., coffee, alcohol), medications (e.g., calcium channel blockers, nitrates, beta-
blockers) & hormones (e.g., progesterone) can decrease the pressure of the LOS.
Obesity is also a contributing factor in GORD, probably because of the increased intra-abdominal pressure.
Large meals (esp fatty) before going to sleep
Smoking
Stress
Typical (oesophageal) symptoms include the heartburn, regurgitation and dysphagia (particularly in the
retrosternal area).
Atypical (extra-oesophageal) symptoms include coughing and/or wheezing (respiratory symptoms resulting from
the aspiration of gastric contents into the tracheobronchial tree or from the vagal reflex arc producing
bronchoconstriction), hoarseness (from irritation of the vocal cords by gastric refluxate and is often experienced
by patients in the morning) and reflux (the most common cause of noncardiac chest pain).
Long standing Gastro-oesophageal reflux disease can cause
oesophagitis
benign oesophageal stricture
Barrett's oesophagus

Q1 - C

Acute appendicitis is sudden inflammation of the appendix, usually caused by obstruction of the lumen resulting in
invasion of the appendix wall by the gut flora.

Most common cause of an acute abdomen in the UK - about 10% of the population will develop acute
appendicitis.
Presentation
Classically periumbilical pain moves to the RIF.
Anorexia, vomiting, constipation, furred tongue, tachycardia, pyrexia 37.5-38.5°C, lying still, foetor
and flushed appearance may occur.
Rovsing's sign is positive when palpation of the lower left quadrant of a person's abdomen results in more
pain in the right lower quadrant indicating the patient may have appendicitis.
Investigations
Appendicitis is essentially a clinical diagnosis but the following may be useful:
Urinalysis may exclude urinary tract infection.
Pregnancy test to exclude ectopic pregnancy
Bloods - mild leukocytosis with raised inflammatory markers but bloods may be normal
Ultrasound may help in some patients where the diagnosis is doubtful and in the assessment of an
appendix mass or abscess.
Management
All suspected cases should be admitted to hospital
Appendicectomy is the treatment of choice and is increasingly done as a laparoscopic procedure.
Complications
Perforation - higher risk in elderly people and young children
Appendix mass - omentum and small bowel adhere to the appendix
usually presents with a fever and a palpable mass

https://www.mcqbank.co.uk/note_manage_print.php Page 40 of 55
06/01/2017, 17)40

Appendix abscess can be shown by ultrasound or CT scan


initial treatment is by percutaneous or open drainage
Pelvic or subphrenic abscess
Adhesions in the longer term may cause intestinal obstruction

Q2 - A

Acute pancreatitis is acute inflammation of the pancreas, releasing exocrine enzymes that cause autodigestion of
the organ.

Causes

Gallbladder disease and excess alcohol consumption account for most cases
“I GET SMASHED”
I Idiopathic
G Gallstones
E Ethanol
T Trauma
S Steroids
M Mumps
A Autoimmune (PAN)
S Scorpion Venom
H Hyperlipidaemia, Hypothermia, Hypercalcaemia
E ERCP
D Drugs:
AIDS drugs DDI and pentamidine
diuretics such as furosemide and hydrochlorothiazide
chemotherapeutic agents L-asparaginase and azathioprine
oestrogen

Presentation

Most commonly, presents as severe upper abdominal pain of sudden onset with vomiting with mild pyrexia
& tachycardia.
In severe cases: gross hypotension, pyrexia, tachypnoea, acute ascites, pleural effusions, body wall staining
around the umbilicus (Cullen's sign) or flanks (Grey Turner's sign) & hypoxaemia.

Investigations

Bloods
Serum amylase >4 x normal, amylase rises within a few hours after onset, returning to normal 3-5 days
after attack.
Imaging
Plain film to exclude other causes of an acute abdomen
CT (helps if diagnosis equivocal & in assessment of complications)
U/S (useful to detect gallstones)

Assessment of severity

Glasgow, Ranson & APACHE scores


≥ 3 of Modified Glasgow Criteria within 48hrs suggest severe Pancreatitis.
Mnemonic = PANCREAS
PaO2 <8kPa
Age >55yrs
Neutrophils (WBC >15x109/l)
Calcium <2mmol/l
Renal Function (Urea >16mmol/l)

https://www.mcqbank.co.uk/note_manage_print.php Page 41 of 55
06/01/2017, 17)40

Enzymes (LDH >600ih/l; AST>200iu/l)


Albumin <32g/l
Sugar (Glucose >10mmol/l)

Management

Mild cases
Manage on a general ward with pain relief (morphine is relatively contra-indicated because of possible
spastic effect on the sphincter of Oddi), IV fluids with nil by mouth, nasogastric tube if severe vomiting.
Severe cases
Treat in ITU or a high dependency unit.
Where there is evidence of significant pancreatic necrosis, IV antibiotics should be given,
Feed with enteral nutrition via a nasogastric tube
Surgery is only required where there is infection and necrosis.

Complications

Pancreatic necrosis - if infected, this trebles mortality risk


Acute fluid collections -the majority will resolve spontaneously
Pancreatic abscess is a collection of pus adjacent to the pancreas, presenting several months after an attack
Acute pseudo-cyst contains pancreatic juice in a wall of fibrous or granulation tissue
Pancreatic ascites occurs when a pseudo-cyst collapses into the peritoneal cavity or major pancreatic duct breaks
down and releases pancreatic juices into the peritoneal cavity.

Prognosis

5% mortality in mild cases


up to 30% mortality in severe cases

Q3 - E

Ruptured Abdominal Aortic Aneurysm has a mortality of 40% if treated and 100% if untreated.

The patient is usually over 50 years old.


Presents with
intermittent or continuous abdominal pain that can radiate to the back, iliac fossae or groins (& hence be
confused with renal colic)
shock
collapse
sometimes an expansile abdominal mass can be palpated.

Alcohol withdrawal starts 10-72 hours after the last drink.

Signs include

autonomic hyperactivity (e.g. tachycardia, hypertension)


tremor
confusion
fits
hallucinations (delirium tremens)

https://www.mcqbank.co.uk/note_manage_print.php Page 42 of 55
06/01/2017, 17)40

A pharyngeal pouch is a pulsion diverticulum of the pharyngeal mucosa through Killian's dehiscence, an area of
weakness between the two parts of the inferior pharyngeal constrictor - the thyropharyngeus and the cricopharyngeus -
at their posterior margin.

Usually presents in elderly men as dysphagia which is characterised by the first mouthful being swallowed
easily but as the pouch becomes filled with food, obstruction of the upper oesophagus occurs.
Further swallowing is prevented and the contents of the pouch are regurgitated.
There is usually a palpable swelling in the neck in two-thirds of cases which may gurgle and there may also be
fits of coughing and episodes of pulmonary infection, especially at night, due to the inhalation of regurgitated
food.
The diagnosis is confirmed by barium swallow.

Endoscopy does not add further diagnostic information and may be dangerous because of the risks of perforation.
The treatment of a pharyngeal pouch is by excision of the pouch combined with myotomy.

Q1 - E

https://www.mcqbank.co.uk/note_manage_print.php Page 43 of 55
06/01/2017, 17)40

Acute cholecystitis follows impaction of a stone in the cystic duct, which may cause continuous epigastric or right
upper quadrant pain, vomiting, fever, local peritonism, or a gallbladder mass.

The main difference from biliary colic is the inflammatory component (local peritonism, fever, raised white cell
count).
If the stone moves to the common bile duct jaundice may occur.
Repeated attacks of acute cholecystitis lead to chronic cholecystitis, in which the walls of the gallbladder become
thickened and scarred, and the gallbladder becomes shrivelled.
Murphy's sign is performed by asking the patient to breathe out and then gently placing the hand below the
costal margin on the right side at the mid-clavicular line (the approximate location of the gallbladder).
The patient is then asked to breathe in.
During inspiration, the abdominal contents are pushed downward and the gallbladder will meet the
examiner’s hand.
If the gallbladder is tender this will cause the patient to stop inspiring due to pain.
This is considered positive but a positive test also requires no pain on performing the manoeuvre on
the patient's left hand side.
Typically, it is positive in cholecystitis, but negative in choledocholithiasis and ascending cholangitis.

Q2 - B

Gall bladder carcinoma is commoner in the Hispanic populations.

It is associated with gallstones in 80% of cases, with the risk of malignancy increasing with the length of time
that gallstones have been present.
The symptoms of gallbladder cancer overlap with the symptoms of gallstones and biliary colic.
Abdominal pain may be of a more diffuse and persistent nature than the classic right upper quadrant pain
of gallstone disease.
Jaundice, anorexia, and weight loss often indicate more advanced disease.
A mass is usually palpable in the right upper quadrant.
The clinical course of gallbladder cancer is often so indolent that at the time of presentation there has
been local invasion, lymph node metastases, and distant metastases, for example to the lungs or bone.
Early tumours may be treated successfully by cholecystectomy in conjunction with wedge resection of the liver
bed plus regional lymphadenectomy.
Surgery may also be of value for small invasive tumours.
In more advanced diseases, radiotherapy and cytotoxic chemotherapy have no significant effect.
Palliative treatment, for example analgesics and sedatives, are the only course available.

Q3 - G

Acute cholangitis is a bacterial infection superimposed on an obstruction of the biliary tree most commonly from a
gallstone.

Usually affects the elderly.


Charcot’s triad consists of fever, RUQ pain, and jaundice.
Treatment:
Most cases are initially treated with empirical antibacterial therapy that will cover Gram-positive and
Gram-negative aerobic bacteria, as well as anaerobic bacteria such as a broad-spectrum second- or third-
generation cephalosporin plus metronidazole.
Timing of biliary drainage depends on the severity of cholangitis, and on the patient's overall clinical state
and response to antibacterial and supportive therapy.
Drainage is achieved by ERCP, PTC or open surgery.
Mortality rate is approximately 5-10%.

https://www.mcqbank.co.uk/note_manage_print.php Page 44 of 55
06/01/2017, 17)40

Gastroesophageal reflux disease (GORD) occurs when the amount of gastric juice that refluxes into the
oesophagus exceeds the normal limit and causes symptoms.

Abnormal gastroesophageal reflux is caused by the functional or mechanical problems with the lower
oesophageal sphincter (LOS).
It is equally common in both sexes and prevalence increases in the over 40 year olds.
Predisposing factors:
Certain foods (e.g., coffee, alcohol), medications (e.g., calcium channel blockers, nitrates, beta-
blockers) & hormones (e.g., progesterone) can decrease the pressure of the LOS.
Obesity is also a contributing factor in GORD, probably because of the increased intra-abdominal pressure.
Large meals (esp fatty) before going to sleep
Smoking
Stress
Typical (oesophageal) symptoms include the heartburn, regurgitation and dysphagia (particularly in the
retrosternal area).
Atypical (extra-oesophageal) symptoms include coughing and/or wheezing (respiratory symptoms resulting from
the aspiration of gastric contents into the tracheobronchial tree or from the vagal reflex arc producing
bronchoconstriction), hoarseness (from irritation of the vocal cords by gastric refluxate and is often experienced
by patients in the morning) and reflux (the most common cause of noncardiac chest pain).
Long standing Gastro-oesophageal reflux disease can cause
oesophagitis
benign oesophageal stricture
Barrett's oesophagus

Q1 - F

Constipation is infrequent or difficult evacuation of faeces.

Causes include:
organic obstruction
painful anal conditions
adynamic bowel
drugs, habit and diet
metabolic imbalance
Hypothyroidism
Hypercalcaemia
Hypokalaemia
Porphyria
Lead poisoning​
Clinical features include infrequent, incomplete evacuation of stools (generally this is taken to be less than
twice a week
However, normal bowel habit can vary tremendously and what is considered constipation for some is
diarrhoea for others!
More significant is the change in bowel habit relative to the patient's normal bowel habit,
Other symptoms may include anorexia and vague abdominal discomfort, diarrhoea (constipation may
cause overflow or spurious diarrhoea, especially in the elderly, when faecal fluid intermittently escapes past
an impacted faecal mass), acute abdominal pain (usually in children), or features of intestinal
obstruction (usually in the elderly).
On examination, there may be mild abdominal tenderness and a faecally-loaded colon on the left side.
There is usually a mass of faeces felt on rectal examination.
In elderly patients, faeces may be impacted higher up and so an empty rectum does not exclude

https://www.mcqbank.co.uk/note_manage_print.php Page 45 of 55
06/01/2017, 17)40

constipation.
In children, constipation is usually diagnosed based on the history and the examination.
If chronic constipation occurs, then more extensive investigation for conditions such as Hirschsprung's
disease is warranted.
Principles of management in adults:
Treat the cause.
Mobilise the patient.
Consider drugs only if above measures fail.
Try to use drugs for short durations only.

Q2 - C

Pseudo-obstruction presents as colonic obstruction but a mechanical cause cannot be found.

The patient is often elderly & bedridden.


The abdomen gradually distends and bowel actions cease.
Note bowel sounds may still remain and even sound obstructive.
There is usually little or no abdominal pain and no tenderness.
Abdominal x-ray shows a similar picture to mechanical obstruction but gas may be seen in the rectum.
Once mechanical obstruction is excluded, management is conservative.

Q3 - D

A volvulus is a complete twisting of a loop of intestine around its mesenteric attachment site.

This can occur at various locations of the GI tract, including stomach, small intestine, caecum, transverse colon,
and sigmoid colon.
Sigmoid volvulus is the most common form of GI tract volvulus.
It is most common in elderly persons (often neurologically impaired) and patients almost always have a history
of chronic constipation.
Predisposing factors include
chronic constipation
high-roughage diet (may cause a long, redundant sigmoid colon)
roundworm infestation
megacolon (often due to Chagas disease).
Peak age >50 yrs.
Torsion is usually anti-clockwise ranging from 180 – 540 degrees.
Presentation is usually with an acutely distended abdomen, colicky pain (often LLQ), and failure to pass flatus
or stool (constipation is prevailing feature).
The abdominal plain film is usually diagnostic which shows an inverted U-shaped appearance of distended
sigmoid loop.

Treatment options include


de-rotation & decompression by barium enema or with rectal tube, colonoscope, or sigmoidoscope if no

https://www.mcqbank.co.uk/note_manage_print.php Page 46 of 55
06/01/2017, 17)40

signs of bowel ischemia or perforation


laparoscopic de-rotation
laparotomy +/- bowel resection

Correct order: B, E, C, A, D

This question tests issues relating to the end of life care.

The best choice here is option B. Local hospital policy will vary, but in many hospitals junior doctors (with a full
GMC licence to practice – i.e. F2 and above) can sign the first part of the DNACPR form which is valid for 48
hours. Obviously it is important to discuss this with the whole team but often on the ward it is not possible to do
this within the appropriate time frame. Not signing the DNACPR form in this case may actually do more
harm than good as CPR would be very unlikely to be successful and extremely traumatic for the patient
described.
Important changes to clinical care should be added contemporaneously to the patient’s notes (option E).
Whilst obviously good practice for the consultant to counter-sign, this is not legally necessary for 24 hours - YOU
have legitimately made the decision and, without the luxury of time, informing the relatives (option C) that
resuscitation will not be attempted would be a greater priority than chasing down the consultant in this instance
(option A).
Note the joint guidance issued by the BMA, the Resuscitation Council (UK) and the RCN in Oct 2014 that
took into account the Court of Appeal judgement in the Tracey vs Cambridge case June 2014:
"Making a decision not to attempt CPR that has no realistic prospect of success does not require the
consent of the patient or of those close to the patient. However there is a presumption in favour of
informing a patient of such a decision. The patient and those close to the patient have no right to insist on
receipt of treatment that is clinically inappropriate. Healthcare professionals have no obligation to offer or
deliver treatment that they believe to be inappropriate."
Handing the job over to the on call doctor is least appropriate (option D).

Haemoptysis is usually not a feature of pulmonary fibrosis.

Common causes of haemoptysis

Infection (60-70% of cases)


Acute Bronchitis
Pneumonia
Staphylococcus aureus
Pseudomonas aeruginosa
Tuberculosis
Fungal organisms (e.g. Aspergillosis)
Influenza
Lung Cancer (20%)

Less common causes of haemoptysis

Cardiovascular causes
Pulmonary venous hypertension
Congestive heart failure

https://www.mcqbank.co.uk/note_manage_print.php Page 47 of 55
06/01/2017, 17)40

Severe mitral stenosis


Pulmonary embolism
Arteriovenous malformation
Pulmonary causes
Bronchiectasis
Airway trauma or foreign body (esp. children)
Lung abscess
Goodpasture's syndrome
Wegener's granulomatosis

This patient is taking a low dose inhaled corticosteroid and is currently at the ‘regular preventer’ stage. British
Thoracic Society (BTS) asthma guidelines advise moving up a stage if he is using more than 3 doses of short acting β2
agonist a week. The next stage would be ‘initial add-on therapy’ of which an inhaled LABA (normally as a combination
inhaler – e.g. Duoresp Spiromax, Symbicort Turbohaler, etc.) is recommended.

Asthma – BTS Guidelines Summary

Diagnosis

The diagnosis of asthma is a clinical one.


Tests influence the probability of asthma but do not prove a diagnosis.
Asthma status and the outcome of diagnostic tests for asthma vary over time.
An historical record of significantly lower FEV1 or PEF during symptomatic episodes compared to
asymptomatic periods provides objective confirmation of obstructive nature of the episodic symptoms.

https://www.mcqbank.co.uk/note_manage_print.php Page 48 of 55
06/01/2017, 17)40

Supported Self Management

All people with asthma (&/or their parents/carers) should be offered self-management education which should
include a written personalised asthma action plan (‘PAAPs’ – can be downloaded from www.asthma.org.uk)
and be supported by regular professional review.
In adults, written personalised asthma action plans may be based on symptoms and/or peak flows. Symptom-
based plans are generally preferable for children.

Management

The aim is control of the disease. Complete control is defined as:


no daytime symptoms
no night time awakening due to asthma
no need for rescue medication
no asthma attacks
no limitations on activity including exercise
normal lung function (in practical terms FEV1 and/or PEF >80% predicted or best)

https://www.mcqbank.co.uk/note_manage_print.php Page 49 of 55
06/01/2017, 17)40

minimal side effects from medication


Approach to management
Start treatment at the level most appropriate to initial severity.
Achieve early control.
Maintain control by:
Increasing treatment as necessary
Before initiating a new drug therapy, practitioners should check adherence with existing
therapies, check inhaler technique and eliminate trigger factors.
Decreasing treatment when control is good
Patients should be maintained at the lowest possible dose of inhaled corticosteroid (ICS)
Reduction in ICS dose should be slow as patients deteriorate at different rates. Reductions
should be considered every 3m, decreasing the dose by about 25-50% each time.
Doses of ICS are expressed as very low (generally paediatric dose), low (generally starting dose for adults),
medium and high.
Adult Management
Link for table of adult doses of ICS

Paediatric Management
Link for table of paediatric doses of ICS

https://www.mcqbank.co.uk/note_manage_print.php Page 50 of 55
06/01/2017, 17)40

Exercise induced asthma

For most patients, exercise-induced asthma is an expression of poorly controlled asthma and regular treatment
including ICS should be reviewed.
If exercise is a specific problem in patients taking ICS who are otherwise well controlled, consider adding one of
the following therapies:
leukotriene receptor antagonists (LTRA) e.g. montelukast
long-acting β2 agonists (LABA) e.g. salmeterol
sodium cromoglicate or nedocromil sodium
oral β2 agonists
theophyllines
Immediately prior to exercise, inhaled short-acting β2 agonists are the drug of choice.

Difficult asthma

Difficult asthma is defined as persistent symptoms and/or frequent asthma attacks despite treatment with
high-dose therapies.
Patients with difficult asthma should be systematically evaluated, including:
confirmation of the diagnosis of asthma
identification of the mechanism of persisting symptoms
assessment of adherence to therapy
This assessment should be facilitated through a dedicated multidisciplinary difficult asthma service.
Be aware that difficult asthma is commonly associated with coexistent psychological morbidity.

References
British Thoracic Society/Scottish Intercollegiate Guidelines Network - British guideline on the management of asthma.

https://www.mcqbank.co.uk/note_manage_print.php Page 51 of 55
06/01/2017, 17)40

Peak expiratory flow is a measure of maximum expiratory flow occurring just after the start of a forced expiration
from the point of maximum inspiration (total lung capacity).

It correlates well with forced expiratory volume in one second (FEV1) and is used as an estimate of airway
calibre.
However, it is dependent not only on airway calibre, but on lung elastic recoil, patient effort and patient
cooperation.
It is used by asthmatics to gauge response to therapy.
Repeated efforts may cause light-headedness.

Q1 - A

Sarcoidosis is a chronic, noncaseating, granulomatous disease of unknown etiology that affects many organs and
tissues, most commonly the lungs and skin.

Commonest in black women aged 20-40 years.


Because sarcoidosis is a multiorgan disease, patients may complain of a variety of symptoms that may complicate
diagnosis.
50% are asymptomatic and detected incidentally on chest x-ray.
Presents with constitutional symptoms such as fever/night sweats, malaise, fatigue & weight loss as well
as organ specific symptoms
virtually every organ system may be involved - 90% involve the lungs
Investigations
Bloods:
FBC may show raised white count/eosinophilia or lymphopenia. Anaemia may also be seen.
Serum calcium is often elevated.
Hypercalcemia is produced by calcitriol hyperactivity of macrophages
Phosphate and alkaline phosphatase may be increased.
If calcium is elevated then 24-hour urinary collection can prove hypercalciuria
U&E may rarely show renal impairment & LFTs can be deranged.
ESR is often raised.
Serum angiotensin-converting enzyme (ACE) levels are elevated in around 60% of patients
with acute disease and reduce in response to treatment or resolution of the disease.
ACE is produced by the granulomata themselves.
It is a relatively nonspecific finding but can help to make the diagnosis in line with the
clinical context and other investigations.
Chest X-ray may show bilateral hilar or paratracheal lymphadenopathy.
High-resolution CT scanning is often used to detect interstitial lung disease.
Spirometry shows reduced lung volumes, impaired gas transfer and restrictive ventilatory defect.
Bronchoalveolar lavage findings show increased lymphocytes, especially raised CD4:CD8 ratio
which can help to clinch diagnosis.
Transbronchial biopsy can demonstrate the presence of non-caseating granulomata, giving a more
accurate diagnosis.
Gallium scanning may be used to detect extrapulmonary disease
ECG (or 24-hour ECG) may show conducting system defects or the effects of hypercalcaemia.
Diagnosis may be achieved by
Biopsy of any suspicious skin lesions or accessible lymph nodes which will show multiple non-caseating
epithelioid granulomata.

https://www.mcqbank.co.uk/note_manage_print.php Page 52 of 55
06/01/2017, 17)40

Kveim-Siltzbach test involves an intradermal injection of a suspension from the spleen or lymph node of a
patient with a confirmed diagnosis of sarcoidosis.
Test results are considered positive if a nodule appears within 2-7 weeks.
The time required for this test, as well as the risk of transmission of new-variant CJD12 make this
procedure unpopular.
Treatment
Between 30 to 70% of patients do not require therapy.
Corticosteroids, most commonly prednisone, have been the standard treatment for many years.
Approximately half of the cases resolve or can be cured within 12–36 months and most within 5
years.
Some cases persist for several decades.

Q2 - B

Idiopathic pulmonary fibrosis (IPF) is a chronic, progressive, fibrosing interstitial lung disease of unknown
etiology characterized by a poor prognosis and no proven effective treatment.

Clinical features consist of


progressive dyspnoea
non-productive cough
diffuse interstitial infiltrates on chest radiographs

honeycombing on high-resolution CT scans


restrictive impairment with reduced gas exchange on pulmonary function test results

Q3 - D

Atypical pneumonia is a term used to describe a form of pneumonia not caused by one of the more traditional
pathogens.

It can be defined as pneumonia without lobar consolidation.


The term "primary atypical pneumonia" is used to explicitly exclude bacterial pneumonia.
However, in recent years, Mycoplasma, Rickettsia, and Chlamydia are now usually considered bacteria,
albeit unusual types
Mycoplasma is the only type of bacteria with no cell wall
Rickettsia and Chlamydia are intracellular parasites, which used to be confused with viruses.
As the conditions caused by these agents have different courses and respond to different treatments, more specific
identification of the pneumonia should be used when possible.
Mycoplasmal pneumonia as described in this case has an insidious onset over days to weeks with non-specific
symptoms such as fatigue, malaise, headache & diarrhoea. Radiographic findings are variable, but
abnormalities are usually more striking than the findings on physical examination.

https://www.mcqbank.co.uk/note_manage_print.php Page 53 of 55
06/01/2017, 17)40

Suspect mycoplasma pneumonia with a 3 week history of a steadily progressive cough.

The patients PaCO2 is lower than normal (normal PaCO2 4.5-6.0kPa; normal PaO2 9.3-13.3kPa) as hypoxia has
stimulated the peripheral chemoreceptors to increase his breathing rate.
The resulting hyperventilation blows off the extra CO2 and causes a respiratory alkalosis.
In obstructive diseases like asthma, both FEV1 and FVC are reduced with the larger decrease in FEV1.
Therefore FEV1/FVC ratio is decreased.
Poor ventilation of affected areas reduces the V/Q ratio and causes hypoxia.
The patient’s residual volume is increased because breathing at a higher lung volume partially offsets the
increased resistance of the airways.

Pulmonary embolism (PE) is a blockage of the pulmonary artery or one of its branches, usually occurring from an
embolized dislodged deep vein thrombus usually from the pelvis or legs.

Venous stasis from long haul flights is a risk factor. Other risk factors include recent surgery, stroke or MI;
disseminated malignancy; thrombophilia; pregnancy, Pill or HRT.
The classic triad of haemoptysis, dyspnoea and chest pain is rarely the case occurring in fewer than 20% of
patients. Signs may include cyanosis, tachypnoea, tachycardia, hypotension, raised JVP or pleural rub.
ECG may show tachycardia, AF, RBBB, right ventricular strain, SIQIIITIII rare. ABG may show ↓PaO2 &
↓PaCO2.
Causes a type I respiratory failure (hypoxia with normal or low PaCO2) where the hypoxia from the ventilation-
perfusion mismatch leads to an increased respiratory drive, blowing off CO2 and a rise in the pH (respiratory
alkalosis). HCO3- is on the low side (kidneys compensate for the low H+ by reducing reabsorption of HCO3- in an
attempt to try and normalise the pH). Other causes of Type I respiratory failure include pneumonia and pulmonary oedema.
Do D-dimers only in those who do not have a high probability of PE as a negative result will exclude need for
imaging. Positive result does not prove diagnosis and imaging required. CXR may show oligaemia or small
pleural effusion. CTPA (or V/Q scan) is investigation of choice.
Tx is at least 3mths of warfarin (LMWH until INR>2)

https://www.mcqbank.co.uk/note_manage_print.php Page 54 of 55
06/01/2017, 17)40

Correct responses: C, E, G

Questions on confidentiality are common.

The DVLA is responsible for deciding if a patient is unfit to drive.


The role of the doctor is to advise the patient that they have a legal duty to inform the DVLA.
If the patent refuses to accept the diagnosis you can arrange a second opinion for them but in the meantime
they should stop driving.
If the patient continues to drive against medical advice, the doctor should make every reasonable
effort to persuade the patient to stop driving.
This may involve discussion with the patients’ relatives, friends or carers if the patient agrees.
In November 2015, following the investigation of a high-profile incident where a bin lorry crashed into
pedestrians in Glasgow in 2014, the GMC published new guidance emphasising a doctor’s duty to disclose
information to the DVLA (or DVA in Northern Ireland), where a patient continues to drive against medical
advice and fails to inform the DVLA/DVA themselves - see http://www.gmc-uk.org/news/27477.asp.
In this scenario, she was seen in your surgery with her husband so he would be aware of the situation.
If the patient still continues to drive the doctor may inform the DVLA after notifying the patient of the
intended action.
Any action taken should be documented in the notes and a letter should be written to the patient
confirming that disclosure has been made.
Notifying the DVLA immediately is incorrect for the reasons stated above.
Disclosure to the police should only be made “in exceptional cases where the benefits to an individual
or to society of the disclosure outweigh the public and the patient’s interest in keeping the
information confidential.
In all cases where you consider disclosing information without consent from the patient, you must
weigh the possible harm (both to the patient, and the overall trust between doctors and patients)
against the benefits which are likely to arise from the release of information”.
Contacting the Insurance Company is inappropriate as is trying to secure video footage.
If you are unsure of any situation a second opinion is always helpful as is discussing difficult cases with
your peer group but in this scenario there were better options.
You may find the following references helpful in understanding the complex field of confidentiality.
http://www.gmc-uk.org/Confidentiality___reporting_concerns_to_the_DVLA_or_DVA.pdf_58821800.pdf
http://www.direct.gov.uk/en/Motoring/DriverLicensing/MedicalRulesForDrivers/index.htm

https://www.mcqbank.co.uk/note_manage_print.php Page 55 of 55

You might also like